Comprehensive Review in Clinical Neurology: A Multiple Choice Question Book for the Wards and Boards

Chapter 2. Vascular Neurology

Question

1. A 65-year-old patient with diabetes presents with a TIA. According to the ABCD2 score that assesses stroke risk in someone with TIA, which of the following is not a predictor of occurrence of a stroke?

a. Diabetes

b. Age of 60 years or more

c. Hypertension

d. Duration of neurologic symptoms

e. Hyperlipidemia

2. A 42-year-old woman with diabetes, hypertension, and hyperlipidemia is brought to the emergency department for unresponsiveness. An MRI is shown in Figure 2.1. Which of the following will most likely be encountered in this patient?

a. Vertical gaze impairment

b. Hemisensory symptoms

c. Hemiparesis

d. Quadriplegia with impaired horizontal gaze

e. Aphasia

3. A 49-year-old man with history of hypertension presents to the emergency department with acute onset of right hemiparesis and aphasia about 45 minutes ago. The National Institutes of Health Stroke Scale (NIHSS) score is 14. Which of the following is the best next step?

a. Start intravenous tissue plasminogen activator (tPA)

b. Get a brain CT scan

c. Give aspirin 325 mg once

d. Start intravenous heparin

e. Get a brain MRI

FIGURE 2.1 Axial diffusion-weighted MRI

4. A 49-year-old woman presents with acute onset of hemiplegia, progressing to quadriplegia over the next 2 hours. On examination she seems awake; however, she is unable to verbalize. She cannot move her eyes horizontally but is able to move them vertically and blink. Which of the following is the most likely cause?

a. Bilateral thalamic infarcts

b. Lateral medullary infarct

c. Top of the basilar occlusion

d. Infarct affecting the base of the pons bilaterally

e. Infarct affecting the dorsal midbrain

5. Which of the following is correct regarding thrombosis of the venous sinuses?

a. Diplopia with a sixth cranial nerve palsy is specific for cavernous sinus thrombosis

b. Headache is present in less than 50% of cases

c. Increased intracranial pressure is uncommon

d. Superior sagittal sinus thrombosis may produce bilateral thalamic venous infarcts

e. Seizures are more common in venous infarcts as compared to arterial infarcts

6. A 51-year-old man presents with ataxia. An MRI is obtained and is shown in Figure 2.2. Which of the following will most likely result from this injury?

Axial diffusion-weighted MRI

a. Vertigo, nystagmus, and right sided ptosis and miosis

b. Vertical gaze impairment

c. Right third and sixth nerve palsies

d. Hemisensory loss to pain and temperature on the left side of the face

e. Hemisensory loss to pain and temperature on the right side of the body

7. A 34-year-old man presents with vertigo and neck pain after riding a “very wild” roller coaster. The examination demonstrates anisocoria with mild ptosis on the left side, nystagmus, and left side ataxia. Which of the following is the best diagnostic test?

a. Carotid ultrasound

b. Transcranial doppler ultrasonography

c. Transthoracic echocardiogram with bubble study

d. Cerebral angiogram

e. Time-of-flight MRA

8. A 69-year-old woman presents to the emergency department at 2 hours and 35 minutes from the onset of left hemiparesis and hemineglect. Her National Institutes of Health Stroke Scale (NIHSS) score is 16. A brain CT scan shows no hemorrhage. Which of the following statements is correct regarding tissue plasminogen activator (tPA)?

a. The risk of hemorrhage with tPA is similar to placebo.

b. Earlier administration carries a better prognosis and a lower risk of hemorrhage.

c. There is no maximum dose.

d. The use of tPA improves short-term but not long-term clinical outcomes.

e. tPA should not be administered beyond 2 hours from the onset of symptoms.

9. A 72-year-old woman with a history of diabetes presents with a transient visual disturbance on the right side. On further questioning, she reports that she experienced a transient dimness of the vision of the right eye, progressing from the upper visual field to the lower visual field like a “shade” falling. This lasted for about 10 minutes and resolved on its own. Which of the following is correct?

a. This patient is having transient ischemia of the left occipital cortex

b. A transcranial doppler will demonstrate increased velocities in the basilar artery

c. The right ICA is stenotic with an atherosclerotic plaque

d. The left ICA is stenotic with an atherosclerotic plaque

e. Fundoscopic examination will demonstrate papilledema

10. A 40-year-old man presents with tinnitus, unilateral hearing loss, nausea, vomiting, and vertigo. On examination, he has nystagmus, ipsilateral ataxia, ipsilateral Horner’s syndrome, and contralateral sensory deficits to pain and temperature of the arm, trunk, and leg. Which of the following is the most likely diagnosis?

a. Anterior inferior cerebellar artery (AICA) stroke

b. Posterior inferior cerebellar artery (PICA) stroke

c. Midbrain infarct

d. Occipital lobe infarct

e. Superior cerebellar infarct

11. A 59-year-old woman presents with altered mental status. There is restricted diffusion on the MRI which is shown in Figure 2.3. Which of the following is the most likely diagnosis?

FIGURE 2.3 Axial diffusion-weighted MRI

a. Bilateral anterior choroidal artery stroke

b. Stroke from occlusion of the recurrent artery of Heubner

c. Stroke from occlusion of the artery of Percheron

d. Stroke from occlusion of the pericallosal artery

e. Superior sagittal sinus thrombosis

12. A 49-year-old woman with anxiety, depression, hypertension, and diabetes presents to the emergency department with a sensory deficit affecting her right face, arm, trunk, and leg, which started yesterday in the evening. The symptoms reached their peak on the morning of presentation. There are no motor deficits on examination. Which of the following is correct?

a. No further work up is needed, and the patient can be discharged from the emergency department

b. Given the lack of motor deficits, her symptoms are most likely related to anxiety

c. The most likely location of the lesion is in the cortex

d. Cardioembolism is the most likely etiology

e. Small vessel disease is the most likely etiology

13. A 52-year-old man with history of smoking, diabetes mellitus, and hypertension presents with acute onset of left hemiparesis affecting the face, arm, and leg. He recognizes the left side of his body and acknowledges the deficits. His gaze is not deviated. Which is the most likely location of the vascular occlusion?

a. Trunk of the right MCA before the bifurcation

b. Right lenticulostriate branches of the MCA

c. Superior division of the right MCA

d. Right PCA

e. Inferior division of the right MCA

14. A 59-year-old woman with a history of atrial fibrillation and hypertension presents with right hemiparesis affecting mainly the face and arm. Her eyes are deviated toward the left. She seems to be able to understand and follow commands, however, cannot verbalize, and appears frustrated when asked questions because she cannot answer. Where is the location of the vascular occlusion?

a. Trunk of the MCA before the bifurcation

b. Lenticulostriate branches of the left MCA

c. Superior division of the left MCA

d. Inferior division of the left MCA

e. Penetrating branches at the level of the pons

15. A 51-year-old man with hypertension and diabetes presents with left leg weakness associated with urinary incontinence. This patient was known to have a normal circle of Willis based on a previous MRA performed for other reasons. Where is the most likely vascular occlusion?

a. Right ACA proximal to the origin of anterior communicating artery

b. Right ACA distal to the origin of the anterior communicating artery

c. Anterior communicating artery

d. Superior division of the right MCA

e. Inferior division of the right MCA

16. Which of the following is correct regarding the recurrent artery of Heubner?

a. It is a penetrating branch of the ACA

b. It is a penetrating branch of the PCA

c. It provides bilateral perfusion to the thalamus

d. Provides perfusion to the posterior limb of the internal capsule

e. Originates from the main trunk of the MCA

17. The lenticulostriate branches provide perfusion to all of the following structures except:

a. Part of the head and body of the caudate

b. Posterior limb of the internal capsule

c. The putamen

d. External part of the globus pallidus

e. Anterior limb of the internal capsule

18. A 31-year-old man presents with dizziness, vertigo, and hoarseness after having chiropractic neck manipulation by an unexperienced practitioner. On examination, the patient has nystagmus, findings of Horner’s syndrome on the right side, paralysis of the right palate, decreased sensation to pinprick on the right side of the face and left hemibody, and right-sided ataxia. Which of the following is the most likely diagnosis?

a. Right medial medullary syndrome

b. Right lateral medullary syndrome

c. Left medial medullary syndrome

d. Left lateral medullary syndrome

e. Right pontine infarct

19. Which of the following options is correct regarding the vascular supply of the thalamus?

a. It is primarily provided by the anterior circulation

b. The anterior choroidal artery supplies the ventral posteromedial nucleus

c. The posterior choroidal artery supplies the ventral anterior nucleus

d. The paramedian branches supply the dorsomedial nucleus

e. The posterior choroidal artery arises from the posterior communicating artery and supplies the pulvinar

20. A 49-year-old man presents with ataxia. His brain CT scan is shown in Figure 2.4. Which of the following is the most likely artery involved?

a. Anterior inferior cerebellar artery (AICA)

b. Superior cerebellar artery (SCA)

c. Posterior inferior cerebellar artery (PICA)

d. PCA

e. Vertebral artery

21. You are asked to see a 42-year-old right-handed woman with diabetes and dilated cardiomyopathy who developed acute “confusion.” On examination, she does not follow commands and is speaking fluently and saying multiple phrases that do not make sense. She seems to have a visual defect in the right hemifield. An MRI is obtained and shows evidence of a stroke. Where is the most likely vascular occlusion?

a. Trunk of the left MCA prior to the bifurcation

b. Lenticulostriate branches of the left MCA

c. Superior division of the left MCA

d. Inferior division of the left MCA

e. Penetrating branches at the level of the pons

FIGURE 2.4 Axial CT

22. What syndrome can you expect in a patient with the MRA shown in Figure 2.5?

a. Wallenberg’s syndrome

b. Parinaud’s syndrome

c. A left MCA syndrome

d. Locked-in syndrome

e. Dejerine-Roussy syndrome

FIGURE 2.5 MRA of the circle of Willis

23. An MRI is shown in Figure 2.6. Which of the following is correct regarding this condition?

FIGURE 2.6 Axial diffusion-weighted MRI

a. Aphasia and neglect are common neurologic manifestations

b. Echocardiogram with bubble study and Holter monitor are indicated

c. Pathologic analysis of this lesion would demonstrate microhemorrhages in the affected area

d. It is caused by occlusion of a penetrating vessel and intimately related to hypertension

e. The most common presentation in this case is pure sensory deficits

24. Which is the abnormality in the MRA shown in Figure 2.7?

FIGURE 2.7 MRA

a. Absence of the left vertebral artery

b. Basilar occlusion

c. Left MCA occlusion

d. Left ICA occlusion

e. Fetal posterior cerebral arteries

25. Which of the following is incorrect regarding the anterior choroidal artery?

a. It is a branch of the ICA

b. It supplies the posterior limb of the internal capsule

c. It supplies the anterior limb of the internal capsule

d. It supplies a part of the geniculocalcarine tract

e. It supplies the choroid plexus in the lateral ventricles

26. A 49-year-old patient presents with acute onset of neurologic symptoms. A cerebral angiogram is shown in Figure 2.8. Which is the abnormality in this angiogram?

FIGURE 2.8 Angiogram

a. Left MCA occlusion

b. Left vertebral artery occlusion

c. ACA occlusion

d. ICA occlusion

e. This is a normal angiogram

27. Which of the following is the most likely mechanism of the stroke shown in Figure 2.9?

FIGURE 2.9 Axial diffusion-weighted MRI

a. Cardioembolic

b. Lacunar strokes

c. Embolic from large artery atherosclerosis

d. Hypotension in the setting of carotid stenosis

e. Venous infarction

28. A 39-year-old woman presents with Horner’s syndrome on the left side, with vertigo, ataxia, and sensory changes on the left side of the face and right side of the body. Which of the following is the most likely cause?

a. Left carotid artery dissection

b. Right carotid artery dissection

c. Left vertebral artery dissection

d. Right vertebral artery dissection

e. Right MCA stroke

29. A 52-year-old woman presents to the emergency department with acute onset of right hemiparesis. A CT scan is obtained and is shown in Figure 2.10. Which of the following is correct?

a. This patient is at risk of vasospasm

b. The cause of the symptoms is a lacunar stroke

c. The patient may need suboccipital craniectomy for decompression

d. The patient has a hypertensive hemorrhage

e. The patient has an MCA occlusion

30. A 59-year-old man with a history of diabetes, hypertension, and hyperlipidemia presents with a blood pressure of 60/30 mm Hg and confusion. His creatine kinase is very elevated with an MB fraction of 12%. His troponin is elevated, and the electrocardiogram shows ST depressions in the inferior leads. After stabilization he is noticed to have left side weakness, more prominent in the shoulder abductors and hip flexors. Which of the following is the most likely underlying mechanism of his weakness?

FIGURE 2.10 Axial CT

a. Weakness from a myopathy

b. Internal capsule lacunar infarction

c. Hypotension in the setting of a left internal carotid stenosis

d. Hypotension in the setting of a right internal carotid stenosis

e. Cardioembolic event associated with arrhythmia

31. A 57-year-old man presents with acute onset of paralysis of the right arm and leg sparing the face, loss of position and vibration sensation on the right side of the body, and dysarthria. On examination, it is also noticed that the tongue deviates to the left. Which of the following is the most likely diagnosis?

a. Right lateral medullary syndrome

b. Left medial medullary syndrome

c. Right medial medullary syndrome

d. Right pontine infarct

e. Left lateral medullary syndrome

32. A 65-year-old woman went on a roller coaster ride. About 3 weeks later, she had an episode of what sounds like amaurosis fugax in the left eye. She complains of headaches, feels weak “all over,” and states that her shoulders have been aching for at least 6 months. She also reports that when eating, she gets tired of chewing and her jaw hurts. Initial noninvasive imaging studies of the brain and intracranial and extracranial circulation are unremarkable. Besides checking an ESR, which of the following is the best next step?

a. Cerebral angiogram

b. CT angiogram of the neck

c. Start heparin and bridge to warfarin

d. Schedule a temporal artery biopsy

e. Start steroids

33. A 49-year-old woman presents with acute onset of right side facial weakness, involving both the upper and lower face, and left hemiplegia. Which of the following is the most likely diagnosis?

a. Right pontine infarct

b. Left pontine infarct

c. Right midbrain infarct

d. Left midbrain infarct

e. Right MCA infarct

34. A 42-year-old woman presents for evaluation of headaches and is found to have a left MCA bifurcation aneurysm. The patient asks about risks. Which of the following is not correct?

a. Patients with previous aneurysmal rupture have a higher risk of SAH

b. Smoking is a risk factor for aneurysmal rupture

c. Anterior circulation aneurysms have a higher risk of rupture when compared with posterior circulation aneurysms

d. Patient age should be taken into consideration when formulating the management strategy

e. Uncontrolled hypertension may be a risk for aneurysmal rupture

35. A 59-year-old right-handed man presents with neurologic deficits. A brain CT scan is obtained and is shown in Figure 2.11. Which of the following manifestations would not be seen in this patient as a result of this lesion?

a. Right homonymous hemianopia

b. Anomia

c. Alexia

d. Visual agnosia

e. Expressive aphasia

36. A 52-year-old woman with diabetes comes to the clinic with a sudden onset of vertical diplopia with limited adduction and vertical movements of the right eye. She also has tremor and choreoathetotic movements on the left side of her body. Which of the following is the most likely diagnosis?

a. Right ventral mesencephalic tegmentum infarct

b. Left ventral mesencephalic tegmentum infarct

c. Right pontine infarct

d. Left pontine infarct

e. Quadrigeminal plate infarct

37. A right-handed patient presents with acute onset of neurologic deficits, and an MRI of the brain is obtained and is shown in Figure 2.12. Which of the following manifestations may be seen in this patient?

FIGURE 2.11 Axial CT

FIGURE 2.12 Axial diffusion-weighted MRI

a. Neglect of the left side of the body

b. Gaze deviation toward the right

c. Global aphasia

d. Paresis of the right leg more than face and arm

e. Left homonymous hemianopsia

38. Which of the following patients should be managed with antiplatelet agents for stroke prevention?

a. A 50-year-old man who suffered an ST-elevation myocardial infarction last week and has an ejection fraction of 30% with anterior wall akinesis and a left ventricular thrombus

b. A 49-year-old woman with a mechanical heart valve

c. A 52-year-old man with hyperthyroidism and intracranial atherosclerotic stenosis

d. A 76-year-old man with atrial fibrillation, diabetes, hypertension, and congestive heart failure

e. A 70-year-old man with an intracardiac thrombus

39. A 49-year-old right-handed man with a history of atrial fibrillation and hypertension presents with acute onset of right hemiparesis affecting the face, arm, and leg. His gaze is deviated toward the left, and he seems to have a right homonymous hemianopsia. He is globally aphasic. What is the location of vascular occlusion?

a. Trunk of the MCA before the bifurcation

b. Left lenticulostriate branches of the MCA

c. Inferior division of the left MCA

d. Penetrating branches at the level of the pons

e. Superior division of the left MCA

40. Which of the following is incorrect regarding the intracranial circulation?

a. The ACA and MCA are branches of the ICA

b. The PCAs are branches of the basilar artery

c. The pericallosal artery arises from the anterior circulation

d. Lenticulostriate branches arise from the PCA

e. The recurrent artery of Heubner arises from the ACA

41. A 50-year-old woman with hypertension presents to the clinic with a history of TIA about a month ago. Aspirin 81 mg was started at that time. Cardioembolic work up was negative, carotid ultrasound demonstrated nonsignificant stenosis, and low-density lipoprotein (LDL) was 110 mg/dL. Which of the following agents has been shown to prevent recurrent cerebrovascular events and should be used in this patient?

a. Statins

b. Warfarin

c. Tissue plasminogen activator (tPA)

d. Heparin

e. Hormone replacement therapy

42. A patient presents with limited upward gaze bilaterally, nystagmus on attempted convergence, and skew deviation. The pupils are fixed with abnormal accommodation and light-near dissociation. Where is the lesion?

a. Ventral midbrain

b. Pons

c. Medulla

d. Bilateral thalami

e. Quadrigeminal plate

43. A 53-year-old woman who underwent coronary artery bypass surgery 5 days ago develops acute onset of right hemiparesis and aphasia. As per the nurse, the patient had shaking of her right arm just prior to the onset of the neurologic deficits. A brain CT scan obtained 30 minutes from the onset of symptoms shows no hemorrhage and no evidence of acute stroke. The patient’s blood pressure is 170/100 mm Hg, and her blood glucose is 48 mg/dL. The National Institutes of Health Stroke Scale (NIHSS) score is initially 16 prior to the CT scan, but at a second evaluation after the scan the NIHSS score is 6. A decision is made not to give her intravenous tPA. Which of the following is not a contraindication to give this therapy?

a. Major surgery within the last 14 days

b. Seizure at the onset

c. Rapidly improving or minor symptoms

d. Glucose less than 50 mg/dL or more than 400 mg/dL

e. CT scan of the brain showing no evidence of acute infarct

44. A 50-year-old man has a history of stroke in the right MCA distribution. The MRA shows a severe stenosis of the right MCA. As initial therapy, which of the following treatment options has evidence to support its use?

a. Surgical bypass

b. Angioplasty

c. Stent placement

d. Warfarin

e. Aspirin

45. A 59-year-old man presents with sudden onset of right hemiparesis and aphasia. The National Institutes of Health Stroke Scale (NIHSS) score is 18. A brain CT scan is obtained and you calculate the Alberta Stroke Program Early CT Score (ASPECTS). Which of the following is correct?

a. A score of 4 supports the use of intravenous tissue plasminogen activator (tPA)

b. Three CT scan cuts are required to calculate this score

c. The maximum score is 20

d. A score of 7 or less is associated with increased dependence and death

e. The minimum score is 3

46. Which of the following is incorrect regarding the ICA?

a. The cervical segment extends from the bifurcation to the carotid canal at the skull base and has no branches

b. The petrous segment is located in the petrous region of temporal bone

c. The ophthalmic artery arises from the ophthalmic segment

d. The cavernous segment runs within the cavernous sinus

e. The anterior choroidal artery arises from the petrous segment

47. A 69-year-old woman with history of a right MCA territory stroke, hypertension, hyperlipidemia, and diabetes comes with recurrent symptoms of left side numbness. A carotid artery ultrasound is obtained to determine if an endarterectomy is indicated. Which of the following is incorrect?

a. A right carotid stenosis of more than 70% is an indication for endarterectomy in this patient

b. A left carotid stenosis of less than 50% is not treated surgically

c. A right carotid occlusion is not treated surgically

d. A right carotid stenosis of 60% will benefit from surgical treatment

e. If the left carotid stenosis is more than 70%, medical treatment is superior to surgery

48. A 61-year-old man with a history of diabetes on insulin, hyperlipidemia, and hypertension presents with transient left hemiparesis without speech problems, lasting for 20 minutes and resolving. Which of the following is incorrect?

a. The patient should be evaluated as soon as possible

b. Stroke work up should be initiated, including noninvasive imaging of the extracranial arteries

c. The risk of stroke is highest in the period immediately following and soon after a TIA

d. A brain MRI will be helpful to rule out brain infarction

e. A period of observation is indicated before thrombolysis, to determine if the patient has a TIA rather than a stroke

49. A 69-year-old man presents with acute neurologic deficits. His brain CT scan is shown in Figure 2.13. Which of the following is the most likely etiology?

FIGURE 2.13 Axial CT

a. Amyloid angiopathy

b. Intracranial aneurysm

c. Anticoagulation

d. Hypertension

e. Sinus venous thrombosis

50. A 14-year-old girl comes for evaluation with a presumptive diagnosis of Moyamoya. Which of the following is incorrect regarding this condition?

a. There is bilateral stenosis of the intracranial internal carotid arteries and other arteries of the circle of Willis

b. There is extensive collateral circulation seen on angiography as a “puff of smoke.”

c. Affects predominately children and adolescents

d. Cerebral ischemia and hemorrhage may occur

e. Anticoagulation has proven to be of benefit in these patients

51. An 82-year-old man presents with an intracranial hemorrhage (ICH) affecting the entire left temporal lobe. He has a history of three ICHs in the past. An MRI obtained a few years ago for evaluation of dementia and prior to his current hemorrhage is shown in Figure 2.14. Which of the following is the most likely diagnosis?

FIGURE 2.14 Axial gradient echo sequence MRI

a. Amyloid angiopathy

b. Intracranial aneurysm

c. Anticoagulation

d. Hypertension

e. Sinus venous thrombosis

52. A 49-year-old woman suffers an acute stroke producing significant sensory loss on the left hemibody. About 3 weeks later, the patient comes back to the clinic and complains of severe painful sensation when touched superficially, as well as deep burning pain in the same region. Which of the following is the most likely diagnosis?

a. Medullary infarct

b. Midbrain infarct

c. Caudate infarct

d. Pontine infarct

e. Thalamic infarct

53. Which of the following is incorrect regarding the posterior circulation?

a. The posterior choroidal arteries arise from the posterior circulation

b. The anterior choroidal arteries arise from the posterior circulation

c. The anterior spinal artery arises from the intracranial vertebral arteries

d. The posterior inferior cerebellar artery (PICA) arises from the vertebral arteries

e. A segment of the vertebral artery runs through the transverse foramina of C5-C6 to C2

54. A 29-year-old woman, smoker, on oral contraceptives, with a history of antiphospholipid antibodies and a recent untreated left middle ear infection, developed right hemiparesis and later became comatose. An MRI is obtained and is shown in Figure 2.15. Which of the following is most likely diagnosis?

FIGURE 2.15 Axial FLAIR MRI

a. Hypertensive hemorrhage

b. PCA stroke with hemorrhagic transformation

c. Hemorrhage associated with amyloid angiopathy

d. Venous infarct with hemorrhage

e. SAH

55. A 49-year-old woman with hypertension presents with a TIA suggestive of ischemia in the territory of the right MCA. Imaging studies suggest a 60% stenosis of the right MCA. Based on available evidence, which of the following treatment plans is least warranted in this patient?

a. Start warfarin

b. Start aspirin

c. Start clopidogrel

d. Start a statin in addition to an antiplatelet agent

e. Start aspirin with dipyridamole.

56. A 61-year-old man presents with acute onset of diplopia and difficulty using his right hand. On examination, he has a left third nerve palsy, right-sided tremors, and ataxia, but no choreoathetotic movements. Which of the following is the most likely diagnosis?

a. Left hemispheric infarct

b. Left pontine infarct

c. Left midbrain infarct

d. Right pontine infarct

e. Right midbrain infarct

57. A 40-year-old woman with history of diabetes and hypertension presents with a severe headache. An MRV is obtained and is shown in Figure 2.16. Which of the following is correct regarding this condition?

FIGURE 2.16 MRV

a. Echocardiogram with bubble study and Holter monitor should be obtained to look for the embolic source

b. Given the history of hypertension and diabetes, further investigation for other prothrombotic causes is not needed

c. A middle ear infection may be the etiological factor

d. Anticoagulation is contraindicated

e. Endovascular therapy plays no role in this disease group

58. A 67-year-old man with history of atrial fibrillation and a recent TIA comes for evaluation. A decision is made to put him on warfarin. Which of the following factors is not routinely taken into account to assess the stroke risk in patients with atrial fibrillation?

a. Age

b. History of congestive heart failure

c. History of hypertension

d. History of a prior stroke

e. History of renal disease

59. Which of the following is incorrect regarding the venous system?

a. Veins of the scalp communicate with the dural venous sinuses via emissary veins

b. Ophthalmic veins drain into the cavernous sinus

c. Cavernous sinus drains to the superior and inferior petrosal sinuses

d. The vein of Labbe is the superior anastomotic vein

e. The vein of Rosenthal is a deep vein

60. A 52-year-old patient with a history of migraines and multiple “mini strokes,” is being treated with aspirin. He presents with a new pure motor stroke, and the MRI shows multiple subcortical lacunes and diffuse white matter changes. Routine stroke work up is negative, and he does not have history of diabetes, hypertension, or hyperlipidemia. He is discharged and gets lost to follow up, coming back about 3 years later for evaluation of dementia. His wife reported that the patient has three siblings who had strokes at young ages, and his father also had strokes and developed dementia early in life. The patient subsequently dies, and an autopsy is performed, with a histopathologic specimen of his brain shown in Figure 2.17. Which of the following is correct regarding this condition?

FIGURE 2.17 Brain specimen. (Courtesy of Dr. Richard A. Prayson.) Shown also in color plates

a. It is autosomal recessive

b. Parkinsonism is a classic feature

c. It is associated with NOTCH3 mutation

d. It is an X-linked disorder

e. Migraine headaches are not associated with this condition

61. A 45-year-old man suffers an ST-elevation myocardial infarction requiring percutaneous transluminal coronary angiography with stent placement in the left anterior descending artery. Subsequently, the patient develops right hemiplegia and diplopia, worse when looking upwards and to the right. The patient has limited adduction and upgaze of the left eye. Which is the most likely diagnosis?

a. Right pontine infarct

b. Left midbrain infarct

c. Left pontine infarct

d. Right midbrain infarct

e. Left internal capsule infarct

62. A brain MRI is shown in Figure 2.18. Which of the following is the most likely diagnosis?

FIGURE 2.18 A: Axial T2-weighted MRI; B: Axial gradient echo sequence MRI

a. Arteriovenous malformation (AVM)

b. Cavernous malformation

c. Dural arteriovenous fistula (DAVF)

d. Venous angioma

e. Capillary telangiectasias

63. Which of the following is incorrect regarding cerebral arterial circulation?

a. The left common carotid artery most commonly arises from the aortic arch

b. The left vertebral artery arises from the left subclavian artery

c. The right common carotid artery arises from the innominate artery

d. The left common carotid artery may arise from the innominate artery in some cases

e. The common carotid artery divides into external and internal carotid arteries at the level of C7

64. A 52-year-old man presents with progressive lower extremity weakness over the past 6 months. He reports that last week he woke up one day and his legs could not move at all. His MRI is shown in Figure 2.19. A vascular anomaly is suspected. Which of the following is the most likely diagnosis?

FIGURE 2.19 Sagittal STIR MRI

a. Epidural hematoma

b. Cavernous malformation

c. DAVF

d. Venous angioma

e. Spinal cord infarct

65. Based on imaging studies shown in Figure 2.20, which of the following is correct?

FIGURE 2.20 (a) Axial CT; (b) Axial T2-weighted MRI; (c) Axial T1-weighted precontrast MRI

a. This is a hyperacute bleed

b. This is not a hemorrhage

c. This is an old hemorrhage, probably more than 1 month old

d. This hemorrhage is about 1 week old

e. This hemorrhage occurred within the last 24 hours

Answer Key

1. e

2. d

3. b

4. d

5. e

6. a

7. d

8. b

9. c

10. a

11. c

12. e

13. b

14. c

15. b

16. a

17. e

18. b

19. d

20. b

21. d

22. d

23. d

24. d

25. c

26. a

27. d

28. c

29. e

30. d

31. b

32. e

33. a

34. c

35. e

36. a

37. c

38. c

39. a

40. d

41. a

42. e

43. e

44. e

45. d

46. e

47. e

48. e

49. d

50. e

51. a

52. e

53. b

54. d

55. a

56. c

57. c

58. e

59. d

60. c

61. b

62. b

63. e

64. c

65. d

Answers

1.e

Hyperlipidemia is not a part of the ABCD2 scoring system.

Identification of risk factors that predict stroke after TIA is important in the assessment of patients presenting with a TIA. The ABCD2 score provides an evaluation of this risk. Points are given for the following risk factors:

Age of 60 years or more: 1 point

Blood pressure of 140/90 mm Hg or greater: 1 point

Clinical symptoms: 1 point for speech impairment without weakness and 2 points for focal weakness

– Symptom Duration: 1 point for 10 to 59 minutes and 2 points for 60 minutes or more

Diabetes: 1 point

The 2-day risk of stroke is 0% for scores of 0 to 1, 1.3% for scores of 2 to 3, 4.1% for scores of 4 to 5, and 8.1% for scores of 6 to 7.

Easton JD, Saver JL, Albers GW, et al. Definition and evaluation of transient ischemic attack. Stroke. 2009; 40:2276–2293.

2. d

The MRI shown in Figure 2.1 demonstrates restriction on DWI of the pons bilaterally, as well as in the cerebellum. Most likely this patient will have quadriplegia and impaired horizontal gaze, findings consistent with locked-in syndrome. Other neurologic manifestations may be present given the extensive stroke in this important structure. However, the other manifestations listed in the options would not be present in this patient.

Goetz CG. Textbook of Clinical Neurology, 3rd ed. Philadelphia, PA: Saunders Elsevier; 2007.

Ropper AH, Samuels MA. Adams and Victor’s Principles of Neurology, 9th ed. New York, NY: McGraw-Hill, 2009.

3. b

A brain CT scan should be obtained before any therapy is started.

This patient’s clinical presentation is consistent with an acute stroke. If this is an acute ischemic stroke, and there are no contraindications, intravenous tissue plasminogen activator (tPA) should be given since the patient presented within the time window for this therapy.

When a patient presents to the emergency department with acute onset of focal neurologic symptoms suggestive of a stroke, and if stable from the hemodynamic and respiratory standpoint, steps should be taken to ensure appropriate treatment in a timely manner. The National Institutes of Health Stroke Scale (NIHSS) is a point system utilized to assess patients presenting with a stroke in which different neurologic functions are examined, including consciousness; vision and eye movement; movement of the face, arm, and leg; sensation; coordination; speech; and language. Points are given for impairments of these functions. The maximum score is 42, with higher scores representing worse neurologic deficits.

The first thing to do is to obtain a brain CT scan to rule out an intracranial hemorrhage (ICH). A brain CT scan will also help to determine if there are other structural lesions and/or already established ischemic changes. If there is no evidence of hemorrhage, an exclusion criteria checklist should be assessed, and if there are no contraindications and if the patient is within the time window, intravenous tPA should be given. The dose is 0.9 mg/kg, with a 10% bolus and the rest over 1 hour, with a maximum dose of 90 mg.

The time window approved on the basis of the National Institute of Neurological Disorders and Stroke (NINDS) tPA trial is 3 hours. However, a recent study (European Cooperative Acute Stroke Study 3 or ECASS3), showed that this medication, when given between 3 and 4.5 hours after the onset of symptoms, can improve clinical outcomes in patients with acute ischemic stroke, with NIHSS score of less than 25 on admission, without exclusion criteria as per the NINDS tPA trial, and with no history of prior clinical stroke or diabetes.

Aspirin may be needed, but a CT scan is a priority to rule out ICH. Intravenous heparin should not be used in the treatment of acute ischemic stroke. A brain MRI is sensitive to detect infarcted tissue, however, is not practical, since it takes time to obtain, and is not widely available in an emergency setting.

Hacke W, Kaste M, Bluhmki E, et al. Thrombolysis with alteplase 3 to 4.5 hours after acute ischemic stroke. N Engl J Med. 2008; 359:1317–1329.

The National Institute of Neurological Disorders and Stroke rt-PA Stroke Study Group. Tissue plasminogen activator for acute ischemic stroke. N Engl J Med. 1995; 335:1581–1587.

4. d

This patient has a locked-in syndrome, which is caused by a basilar occlusion leading to bilateral lesions at the base of the pons, affecting the long tracts but preserving the reticular activating system. These patients are awake, consciousness is preserved, and they can blink and move their eyes vertically; however, they are quadriplegic, unable to speak, and with impairment of horizontal eye movements.

Top of the basilar syndrome results from occlusion at the top of the basilar artery causing infarcts of various structures including the midbrain, thalamus, and temporal and occipital lobes. The manifestations are complex and varied, including combinations of behavioral abnormalities, alteration of consciousness, pupillary manifestations, disorders of ocular movements, visual field defects, and motor and/or sensory deficits.

Infarcts in the other locations will not produce the clinical manifestations that this patient has.

Goetz CG. Textbook of Clinical Neurology, 3rd ed. Philadelphia, PA: Saunders Elsevier; 2007.

Ropper AH, Samuels MA. Adams and Victor’s Principles of Neurology, 9th ed. New York, NY: McGraw-Hill; 2009.

5. e

Seizures occur in 40% of the patients with venous sinus thrombosis, which is a higher percentage than that in patients with arterial strokes. In patients with venous sinus thrombosis, headache is the most frequent symptom, seen in about 90% of cases in adults. Given the occlusion of venous drainage along with hemorrhagic infarct and edema, patients may develop increased intracranial pressure. Diplopia caused by a sixth cranial nerve palsy has nonlocalizing value and may be a manifestation of increased intracranial pressure.

In patients with venous infarcts, focal neurologic findings will be present depending on the area affected along the thrombosed venous sinus. Thrombosis of the deep venous system may lead to deep venous infarcts, including bilateral thalamic infarcts. This is not seen with superior sagittal sinus thrombosis, which instead can lead to infarcts in the parasagittal cortex bilaterally along the sinus.

Ropper AH, Samuels MA. Adams and Victor’s Principles of Neurology, 9th ed. New York, NY: McGraw-Hill; 2009.

Stam J. Thrombosis of the cerebral veins and sinuses. N Engl J Med. 2005; 352:1791–1798.

6. a

This MRI demonstrates restricted diffusion in the lateral medulla and cerebellum, which manifests with Wallenberg’s syndrome. A lateral medullary infarct is caused by occlusion of the posterior inferior cerebellar artery (PICA), and often from occlusion of the vertebral artery.

Wallenberg’s syndrome, or lateral medullary syndrome, involves the following structures:

– Vestibular nuclei, causing vertigo, nystagmus, nausea, and vomiting.

– Descending tract and nucleus of the fifth cranial nerve, producing impaired sensation on the ipsilateral hemiface.

– Spinothalamic tract, producing loss of sensation to pain and temperature in the contralateral hemibody.

– Sympathetic tract, manifesting with ipsilateral Horner’s syndrome with ptosis, miosis, and anhidrosis.

– Fibers of the ninth and tenth cranial nerves, presenting with hoarseness, dysphagia, ipsilateral paralysis of the palate and vocal cord, and decreased gag reflex.

– Cerebellum and cerebellar tracts, causing ipsilateral ataxia and lateropulsion.

– Nucleus of the tractus solitarius, causing loss of taste.

Patients may present with combinations of these manifestations and not always with a complete syndrome. Other clinical manifestations, such as hiccups, are typically seen in this syndrome, but may not be explained by a lesion to a specific structure in the brainstem.

The other manifestations are unlikely to result from the anatomic localization of this infarct.

Goetz CG. Textbook of Clinical Neurology, 3rd ed. Philadelphia, PA: Saunders Elsevier; 2007.

Park MH, Kim BJ, Koh SB, et al. Lesional location of lateral medullary infarction presenting hiccups (singultus). J Neurol Neurosurg Psychiatry. 2005; 76:95–98.

Ropper AH, Samuels MA. Adams and Victor’s Principles of Neurology, 9th ed. New York, NY: McGraw-Hill; 2009.

7. d

This patient likely has a left vertebral artery dissection causing a lateral medullary syndrome. The gold standard diagnostic test for an arterial dissection is a conventional angiogram, demonstrating the narrowing of the vessel, the extension of the dissection with an intimal flap, or double lumen.

Ultrasonography is noninvasive and is useful in the initial assessment; however, the dissection may not be visualized, and ultrasound may not be able to determine the extent of the dissection.

Transcranial Doppler ultrasonography is less useful and does not provide direct visualization of the dissection.

An echocardiogram with bubble study is not indicated.

MRA with a time-of-flight sequence may also be helpful to assess the flow at the site of the dissection; however, it does not provide information about the vessel wall. An MRI with fat-suppression techniques is helpful to assess the vessel wall and surrounding tissues, and very useful in nonocclusive dissections, when conventional angiogram will not give information about the vessel wall.

Patients with arterial dissections will have strokes either from vessel occlusion or from embolism originating in the dissected vessel wall. Treatment ranges from antiplatelet agents to anticoagulation, and in some cases endovascular techniques to open the vessel. However, there is no evidence to dictate which treatment is the best option.

Ropper AH, Samuels MA. Adams and Victor’s Principles of Neurology, 9th ed. New York, NY: McGraw-Hill; 2009.

Schievink WI. Spontaneous dissection of the carotid and vertebral arteries. N Engl J Med. 2001; 344:898–906.

8. b

Tissue plasminogen activator (tPA) is a thrombolytic agent that is used intravenously for the treatment of acute ischemic stroke. Earlier administration correlates with better prognosis and lower risk of hemorrhage when compared with later administration of the medication.

The tPA for acute ischemic stroke trial was published in 1995 and performed by the National Institute of Neurological Disorders and Stroke (NINDS) Study Group. In this trial, intravenous t-PA was given to eligible patients with acute ischemic stroke within 3 hours from onset of symptoms. These patients were given 0.9 mg/kg of the drug, 10% as a bolus and the rest over 1 hour. The maximum dose is 90 mg.

Symptomatic intracranial hemorrhage (ICH) occurred in 6.4% of the patients who received intravenous tPA, compared with 0.6% in those who received placebo. The time window from symptom onset to allowable time for tPA administration was 3 hours; however, it was determined that the earlier the administration, the better the prognosis and the lower the risk of hemorrhage.

Patients who received intravenous tPA had improved clinical outcomes and were at least 30% more likely to have minimal or no disability at 3 months. The mortality at 3 months was 17% in the tPA group and 21% in the placebo group.

Administration of intravenous tPA is indicated in patients with an acute ischemic stroke presenting within 3 hours from symptom onset when no contraindications are identified.

Based on the European Cooperative Acute Stroke Study 3 (ECASS3), the use of intravenous tPA between 3 and 4.5 hours from the onset of symptoms has been shown to be beneficial in select patients. However, 4 additional exclusion criteria exist: National Institutes of Health Stroke Scale (NIHSS) 25 or greater, age >80, history of both stroke and diabetes, and any anticoagulant use, regardless of prothrombin time or INR.

Hacke W, Kaste M, Bluhmki E, et al. Thrombolysis with alteplase 3 to 4.5 hours after acute ischemic stroke. N Engl J Med. 2008; 359:1317–1329.

The National Institute of Neurological Disorders and Stroke rt-PA Stroke Study Group. Tissue plasminogen activator for acute ischemic stroke. N Engl J Med. 1995; 335:1581–1587.

9. c

This patient had transient monocular blindness or amaurosis fugax, which may represent atherosclerotic stenosis of the ipsilateral ICA, in this case the right side.

The retinal artery originates from the ophthalmic artery, which is a branch of the ICA. Transient occlusion of the retinal or ophthalmic arteries may manifest as amaurosis fugax.

Amaurosis fugax is often described as a painless visual loss: a “shade” or “curtain” moving in the vertical plane, with a rapid onset and brief duration of few minutes. Vision is most commonly recovered completely; however, the presentation of amaurosis fugax in a patient with an underlying ICA stenosis, may herald the occurrence of a stroke. For this reason, when a patient complains of amaurosis fugax, investigations should be obtained to rule out ICA stenosis. Other more rare but potential causes include giant cell arteritis, and embolism.

The other options listed do not correlate with the presentation of amaurosis fugax.

Goetz CG. Textbook of Clinical Neurology, 3rd ed. Philadelphia, PA: Saunders Elsevier; 2007.

Ropper AH, Samuels MA. Adams and Victor’s Principles of Neurology, 9th ed. New York, NY: McGraw-Hill; 2009.

10. a

This patient has an infarct in the distribution of the anterior inferior cerebellar artery (AICA). The clinical presentation is very similar to that of a posterior inferior cerebellar artery (PICA) stroke or Wallenberg’s syndrome; however, the difference is the ipsilateral deafness that occurs with AICA infarcts, as a consequence of involvement of the lateral pontomedullary tegmentum. By imaging, AICA infarcts affect the cerebellum more ventrally as compared with PICA infarcts.

Infarcts in the other locations will not produce the clinical manifestations depicted in this case.

Goetz CG. Textbook of Clinical Neurology, 3rd ed. Philadelphia, PA: Saunders Elsevier; 2007.

Ropper AH, Samuels MA. Adams and Victor’s Principles of Neurology, 9th ed. New York, NY: McGraw-Hill; 2009.

11. c

Figure 2.3 shows bilateral thalamic infarction, which can be seen with occlusion of the artery of Percheron.

The P1 segment of the PCA gives rise to interpeduncular branches that will provide vascularization to the medial thalamus. Most frequently, these branches arise from each PCA separately and will give perfusion to the thalamus on its respective side. In some cases, a single artery called the artery of Percheron will arise from the P1 segment on one side and will supply the medial thalami bilaterally. This is a normal variant. If an occlusion of the artery of Percheron occurs, the result will be an infarct in the medial thalamic structures bilaterally.

The anterior choroidal arteries do not supply the medial thalamus. The recurrent artery of Heubner is a branch of the ACA that supplies the anterior limb of the internal capsule, the inferior part of the head of the caudate and the anterior part of the globus pallidus.

The pericallosal artery is one of the subdivisions of the ACA, running along the corpus callosum, and does not supply the thalamus.

Thrombosis of the deep venous structures may produce venous infarcts in the thalamus, but this is not seen with superior sagittal sinus thrombosis.

Goetz CG. Textbook of Clinical Neurology, 3rd ed. Philadelphia, PA: Saunders Elsevier; 2007.

Ropper AH, Samuels MA. Adams and Victor’s Principles of Neurology, 9th ed. New York, NY: McGraw-Hill; 2009.

12. e

This patient presents with pure sensory symptoms, which can be seen in a pure sensory lacunar infarct. A lacunar stroke occurs from occlusion of small penetrating arteries, as a consequence of chronic hypertension. Diabetes and hyperlipidemia also play a role but to a lesser degree. These small vessels develop lipohyalinosis with vessel wall degeneration and luminal occlusion. Atherosclerosis of the parent vessel may occlude the opening of these small penetrating branches, or predispose to the entry of embolic material that will occlude them.

These lacunes occur in the putamen, caudate nuclei, thalamus, basis pontis, internal capsule, and deep hemispheric white matter. Symptoms will depend on the location, and several syndromes have been reported including the following:

– Pure motor, usually involving face, arm, and leg equally, and the most frequent location is in the territory of the lenticulostriate branches, affecting the posterior limb of the internal capsule, but has also been described with lacunes in the ventral pons.

– Pure sensory, with hemisensory deficit involving the contralateral face, arm, trunk, and leg from a lacune in the thalamus.

– Clumsy-hand dysarthria occurs more frequently from a lacune in the paramedian pons contralateral to the clumsy hand, but it may also occur from a lacune in the posterior limb of the internal capsule.

– Ataxic-hemiparesis, in which the ataxia is on the same side of the weakness, but out of proportion to the weakness, and this occurs from lacunes in the pons, midbrain, internal capsule, or parietal white matter.

The clinical manifestations of these lacunar infarcts may have a sudden onset; however, it is not infrequent to see a stepwise “stuttering” progression of the neurologic deficits over minutes, and sometimes over hours to even days.

This patient should undergo evaluation, and MRI with diffusion-weighted sequences should be obtained. Even though diabetes control is important, hypertension plays a major role in the pathogenesis of lacunar strokes, and strict blood pressure control may prevent further events.

Ropper AH, Samuels MA. Adams and Victor’s Principles of Neurology, 9th ed. New York, NY: McGraw-Hill; 2009.

13. b

This patient has a lacunar pure motor stroke syndrome. The infarct is most likely in the posterior limb of the internal capsule on the right side, from occlusion of one or many of the lenticulostriate branches of the MCA. These lenticulostriate branches provide vascular supply to the putamen, part of the head and body of the caudate nucleus, the outer globus pallidus, the posterior limb of the internal capsule, and the corona radiata.

Occlusion of the trunk of the MCA will, in addition to weakness, cause other sensory deficits and cortical findings, such as hemineglect, particularly with nondominant strokes. Strokes in the territory of the divisions of the MCA will not present with a pure motor syndrome, and the motor findings will affect the face and arm more than the leg.

A PCA stroke will present with visual and sensory deficits.

Blumenfeld H. Neuroanatomy through Clinical Cases, 1st ed. Sunderland, MA: Sinauer Associates; 2002.

Goetz CG. Textbook of Clinical Neurology, 3rd ed. Philadelphia, PA: Saunders Elsevier; 2007.

Ropper AH, Samuels MA. Adams and Victor’s Principles of Neurology, 9th ed. New York, NY: McGraw-Hill; 2009.

14. c

This patient has an infarct in the distribution of the superior division of the left MCA. There is hemiparesis affecting mainly arm and face, probably from ischemia to the lateral hemispheric surface. Eye deviation toward the left occurs from unopposed action originating from the right frontal eye fields, given that the left frontal eye fields are dysfunctional. This patient has a Broca’s or motor aphasia, in which she is able to understand and follow commands but cannot verbalize. It is common for these patients to seem frustrated since they can understand and know what they want to say but cannot speak. Broca’s aphasia occurs from ischemia in the territory of the superior division of the MCA affecting the dominant inferior frontal gyrus. On the other hand, ischemia in the territory of the inferior division of the MCA in the dominant hemisphere will cause a Wernicke’s rather than a Broca’s aphasia.

A left MCA trunk occlusion will likely produce a global aphasia, and will also produce ischemia in the lenticulostriate arteries territory, therefore presenting with hemiparesis or hemiplegia affecting face, arm, and leg.

An infarct in the territory of the lenticulostriate branches will not present with cortical findings such as aphasia. A pontine stroke will not produce a brachiofacial hemiparesis with motor aphasia.

Blumenfeld H. Neuroanatomy through Clinical Cases, 1st ed. Sunderland, MA: Sinauer Associates; 2002.

Goetz CG. Textbook of Clinical Neurology, 3rd ed. Philadelphia, PA: Saunders Elsevier; 2007.

Ropper AH, Samuels MA. Adams and Victor’s Principles of Neurology, 9th ed. New York, NY: McGraw-Hill; 2009.

15. b

This patient has ischemia in the territory of the right ACA. Given that both ACAs communicate via the anterior communicating artery, an occlusion proximal to the anterior communicating artery may not produce significant clinical manifestations. Therefore, to produce symptoms, the occlusion must occur in the segment distal to the anterior communicating artery.

An infarction occurring from an ACA occlusion distal to the anterior communicating artery presents with contralateral sensorimotor deficits of the lower extremity, sparing the arm and face. There may be urinary incontinence due to involvement of the medial micturition center in the paracentral lobule; sometimes deviation of the eyes to side of the lesion and paratonic rigidity occur.

An anterior communicating artery occlusion does not produce clinical manifestations in patients with otherwise normal perfusion dynamics.

An MCA distribution infarct would produce manifestations in the arm and face rather than in the lower extremity.

Blumenfeld H. Neuroanatomy Through Clinical Cases, 1st ed. Sunderland, MA: Sinauer Associates; 2002.

Goetz CG. Textbook of Clinical Neurology, 3rd ed. Philadelphia, PA: Saunders Elsevier; 2007.

Ropper AH, Samuels MA. Adams and Victor’s Principles of Neurology, 9th ed. New York, NY: McGraw-Hill; 2009.

16. a

The ACA supplies the anterior three quarters of the medial surface of the frontal lobe. Proximal to the circle of Willis and prior to the anterior communicating artery, deep penetrating branches come off the ACA segment of the anterior communicating artery, the recurrent artery of Heubner being the largest of these deep branches. These branches supply the anterior limb of the internal capsule, the inferior part of the head of the caudate nucleus, and the anterior part of the globus pallidus.

Ropper AH, Samuels MA. Adams and Victor’s Principles of Neurology, 9th ed. New York, NY: McGraw-Hill; 2009.

17. e

The MCA has superficial and deep branches. The deep penetrating or lenticulostriate branches supply the putamen, part of the head and body of the caudate nucleus, the external part of the globus pallidus, the posterior limb of the internal capsule, and the corona radiata. The anterior limb of the internal capsule is supplied by the recurrent artery of Heubner and deep branches coming from the ACA.

The superficial branches of the MCA supply the lateral convexity, including the lateral and inferior parts of the frontal lobe, parietal lobe, superior parts of the temporal lobe, and insula.

Ropper AH, Samuels MA. Adams and Victor’s Principles of Neurology, 9th ed. New York, NY: McGraw-Hill; 2009.

18. b

This patient has Wallenberg’s syndrome, or a lateral medullary syndrome, in this case caused by an infarct on the right side. Based on the history, the etiology is likely secondary to vertebral artery dissection. In this patient, the structures affected are the vestibular nuclei, the descending sympathetic tract on the right side, the right fifth, ninth, and tenth cranial nerve nuclei, right cerebellum and/or its connections, and right spinothalamic tract affecting the sensation on the contralateral side. These findings are consistent with an infarct in the lateral medulla on the right side.

Wallenberg’s syndrome, or lateral medullary syndrome, involves the following structures:

– Vestibular nuclei, causing vertigo, nystagmus, nausea, and vomiting.

– Descending tract and nucleus of the fifth cranial nerve, producing impaired sensation on the ipsilateral hemiface.

– Spinothalamic tract, producing loss of sensation to pain and temperature in the contralateral hemibody.

– Sympathetic tract, manifesting with ipsilateral Horner’s syndrome with ptosis, miosis, and anhidrosis.

– Fibers of the ninth and tenth cranial nerves, presenting with hoarseness, dysphagia, ipsilateral paralysis of the palate and vocal cord, and decreased gag reflex.

– Cerebellum and cerebellar tracts, causing ipsilateral ataxia and lateropulsion.

– Nucleus of the tractus solitarious, causing loss of taste.

Patients may present with combinations of these manifestations and not always with a complete syndrome. Other clinical manifestations, such as hiccups, are typically seen in this syndrome but may not be explained by a lesion to a specific structure in the brainstem.

Goetz CG. Textbook of Clinical Neurology, 3rd ed. Philadelphia, PA: Saunders Elsevier; 2007.

Ropper AH, Samuels MA. Adams and Victor’s Principles of Neurology, 9th ed. New York, NY: McGraw-Hill; 2009.

19. d

The vascular supply of the thalamus originates mainly from the posterior circulation.

There are four major arteries supplying four regions of the thalamus:

1. The tuberothalamic artery, also known as polar artery, originates from the posterior communicating artery and supplies the anterior portion of the thalamus, especially the ventral anterior nucleus.

2. The thalamoperforating or paramedian artery originates from the P1 segment of the PCA and supplies the medial aspect of the thalamus, especially the dorsomedial nucleus.

3. The thalamogeniculate artery originates from the P2 segment of the PCA and supplies the lateral aspect of the thalamus, including the ventral lateral group of nuclei.

4. The posterior choroidal artery arises from the P2 segment of the PCA and provides vascularization to the posterior aspect of the thalamus, where the pulvinar is located.

The anterior choroidal artery does not supply the thalamus.

Carrera E, Bogousslavsky J. The thalamus and behavior. Effects of anatomically distinct strokes. Neurology. 2006; 66:1817–1823.

20. b

Figure 2.4 shows an infarct that involves mainly the right cerebellar hemisphere superiorly, consistent with a superior cerebellar artery (SCA) stroke. The CT scan shows a slice at the level of the midbrain; therefore, it is certain that this is the superior cerebellum.

The SCA supplies most of the superior half of the cerebellar hemisphere, including the superior vermis, the superior cerebellar peduncle, and part of the upper lateral pons.

The anterior inferior cerebellar artery (AICA) supplies the inferolateral pons, middle cerebellar peduncle, and a strip of the ventral cerebellum between the posterior inferior cerebellar and superior cerebellar territories.

The posterior inferior cerebellar artery (PICA) supplies the lateral medulla, most of the inferior half of the cerebellum and the inferior vermis.

A PCA lesion will not produce cerebellar strokes.

A vertebral artery lesion may account for PICA strokes but not strokes in SCA territory.

Blumenfeld H. Neuroanatomy through Clinical Cases, 1st ed. Sunderland, MA: Sinauer Associates; 2002.

21. d

This patient has an infarct in the inferior division of the MCA, which supplies the superior parietal and posterior temporal regions. The patient has clinical findings suggestive of a Wernicke’s (receptive) aphasia, in which the patient speaks fluently but what she says does not make sense, and she is not able to understand spoken language or follow commands. This occurs from ischemia to the posterior aspect of the superior temporal gyrus. Patients with ischemia in the territory of the inferior division of the MCA will also present with agitation and confusion, cortical sensory deficits in the face and arm, as well as visual defects in the contralateral hemifield.

An MCA trunk occlusion will affect not only the cortex but also the deep subcortical structures provided by lenticulostriate branches, therefore presenting with cortical findings as well as a dense hemiparesis.

A stroke in the territory of the superior division of the MCA will manifest with an expressive rather than a receptive aphasia.

Stroke in the territory of the lenticulostriate branches will not present with cortical findings. A pontine stroke will not produce an aphasia.

Blumenfeld H. Neuroanatomy Through Clinical Cases, 1st ed. Sunderland, MA: Sinauer Associates; 2002.

Goetz CG. Textbook of Clinical Neurology, 3rd ed. Philadelphia, PA: Saunders Elsevier; 2007.

Ropper AH, Samuels MA. Adams and Victor’s Principles of Neurology, 9th ed. New York, NY: McGraw-Hill; 2009.

22. d

The most prominent finding in the MRA shown in Figure 2.5 is the absence of a basilar artery, probably from an occlusion. A pontine stroke from the basilar occlusion will likely result in a locked-in syndrome. Basilar occlusion may occur from local thrombosis of the basilar artery itself, thrombosis of both vertebral arteries, or thrombosis of a single vertebral artery when it is the dominant vessel. Embolism can occur as well, frequently lodging distally in the vessel.

Wallenberg’s syndrome occurs from an infarct of the lateral medulla, and is usually caused by involvement of the posterior inferior cerebellar artery (PICA), or the parent vertebral artery.

Parinaud’s syndrome is characterized by supranuclear paralysis of eye elevation, defect in convergence, convergence-retraction nystagmus, light-near dissociation, lid retraction, and skew deviation of the eyes. The lesion is localized in the dorsal midbrain and is classically seen with pineal tumors compressing the quadrigeminal plate; however, it can occur from midbrain infarcts.

This patient does not have significant disease of the left MCA.

Dejerine-Roussy syndrome is caused by thalamic infarct, and even though this patient may have lesions in the thalamus, other manifestations are more likely to predominate.

Goetz CG. Textbook of Clinical Neurology, 3rd ed. Philadelphia, PA: Saunders Elsevier; 2007.

Ropper AH, Samuels MA. Adams and Victor’s Principles of Neurology, 9th ed. New York, NY: McGraw-Hill; 2009.

23. d

The MRI shown in Figure 2.6 demonstrates a very small area of restricted diffusion consistent with a lacunar stroke, which is caused by the occlusion of a small penetrating vessel. This condition is intimately related to hypertension.

The pathologic basis of lacunes is lipohyalinosis of small penetrating branches but not microhemorrhages.

Given that this is a disease of small vessels, studies to look for a cardioembolic source are not required.

Given the location of the lacune in this case, it is unlikely for this patient to present with a pure sensory syndrome, which will be seen more frequently with thalamic lacunes.

Aphasia and neglect are manifestations of infarcts affecting the cortex and are not seen with subcortical strokes.

Ropper AH, Samuels MA. Adams and Victor’s Principles of Neurology, 9th ed. New York, NY: McGraw-Hill; 2009.

24. d

The MRA shown in Figure 2.7 demonstrates absence of the ICA on the left side. The left MCA is still partially seen and is being supplied via the anterior communicating artery.

Both vertebral arteries are seen, and the basilar artery is also present, branching at its top into both PCAs. Fetal PCAs originate from the anterior circulation (distal internal carotid arteries) and not from the basilar artery, and this can be seen in the normal population. This is not the case in this MRA.

Ropper AH, Samuels MA. Adams and Victor’s Principles of Neurology, 9th ed. New York, NY: McGraw-Hill; 2009.

25. c

The anterior choroidal artery does not supply the anterior limb of the internal capsule, which is supplied by the recurrent artery of Heubner and deep penetrating branches of the ACA.

The anterior choroidal artery arises from the ICA just above the origin of the posterior communicating artery, and supplies the internal segment of the globus pallidus, part of the posterior limb of the internal capsule, and part of the geniculocalcarine tract. As it penetrates the temporal horn of the lateral ventricle, it supplies the choroid plexus and then joins the posterior choroidal artery from the posterior circulation.

Ropper AH, Samuels MA. Adams and Victor’s Principles of Neurology, 9th ed. New York, NY: McGraw-Hill; 2009.

26. a

Figure 2.8 shows a conventional angiogram demonstrating occlusion of the main trunk of the left MCA, which is not filling with contrast. The ICA is visualized up to its terminus, and the ACA is also well visualized. In this case, the injection of contrast was performed on the left ICA, and therefore the vertebral artery cannot be assessed.

Ropper AH, Samuels MA. Adams and Victor’s Principles of Neurology, 9th ed. New York, NY: McGraw-Hill; 2009.

27. d

Figure 2.9 demonstrates restricted diffusion almost linearly between vascular boundaries, consistent with a watershed infarction, in this case between the superficial and deep territories of the MCA. Watershed infarcts occur when there is reduction of blood supply to two vascular territories; these regions are most susceptible to ischemia. This reduction of blood flow can occur in the setting of systemic hypotension, especially with an underlying stenosis proximal to both territories, in this case, hypotension and a left carotid stenosis.

The distribution of the stroke in the MRI is consistent with a watershed infarct, and not likely an embolic stroke, lacunar strokes, or venous infarct.

Blumenfeld H. Neuroanatomy Through Clinical Cases, 1st ed. Sunderland, MA: Sinauer Associates; 2002.

28. c

This patient has left-sided Horner’s syndrome as well as findings suggesting a left fifth cranial nerve lesion, left cerebellar, and vestibular nuclei involvement. This can occur at the level of the brainstem, more specifically in the lateral medulla. Therefore, the most likely cause is a left vertebral artery dissection.

A left carotid artery dissection may cause left-sided Horner’s syndrome by affecting the fibers running along the carotid artery walls but will not explain brainstem findings depicted in this case. A right MCA stroke will not produce these clinical manifestations.

Goetz CG. Textbook of Clinical Neurology, 3rd ed. Philadelphia, PA: Saunders Elsevier; 2007.

Ropper AH, Samuels MA. Adams and Victor’s Principles of Neurology, 9th ed. New York, NY: McGraw-Hill; 2009.

29. e

Figure 2.10 shows a hyperdense left MCA sign. In a patient with a presumed stroke, the hyperdense MCA sign has good specificity and positive predictive value for atheroembolic occlusions of the affected vessel, and it is associated with poor prognosis. This sign lacks sensitivity but is helpful when a strong clinical suspicion exists. Mimics of hyperdense MCA sign, also known as pseudohyperdense sign, include vascular calcification, increased hematocrit, and intravenous contrast.

Other early signs (within 6 hours) of ischemic stroke on CT scan include the loss of the insular ribbon, the attenuation of the lentiform nucleus, and the hemispherical sulcal effacement.

This patient does not have a hypertensive hemorrhage. There is no evidence on CT scan of SAH to suspect the development of vasospasm. An MCA occlusion is a large vessel occlusion and does not represent a lacunar stroke. Large MCA occlusions with large strokes may require hemicraniectomy, but not suboccipital craniectomy, which is used in cerebellar strokes.

Jha B, Kothari M. Pearls & Oysters: Hyperdense or pseudohyperdense MCA sign. A Damocles sword? Neurology. 2009; 72:e116–e117.

Koga M, Saku Y, Toyoda K, et al. Reappraisal of early CT signs to predict the arterial occlusion site in acute embolic stroke. J Neurol Neurosurg & Psychiatry. 2003; 74:649–653.

30. d

This patient suffered a watershed infarct affecting the right hemisphere, likely secondary to reduction of blood supply from hypotension in the setting of a right internal carotid stenosis.

Watershed infarcts manifest clinically with proximal weakness, affecting the proximal upper and proximal lower extremities, with weakness at the shoulder and at the hip. This occurs because the watershed regions correlate with the homuncular representation of the proximal limbs and trunk. In severe cases of bilateral watershed infarcts, a “person-in-a-barrel” syndrome occurs, in which the patient can only move the distal extremities.

The clinical history of hypotension and subsequent neurologic findings correlates with watershed ischemic events.

Given the unilateral involvement and the clinical history, this is not a myopathy. An infarct in the internal capsule will not produce the clinical picture that this patient has. A cardioembolic event is also less likely given the clinical presentation. Given the left side weakness, it is likely that there is a stenosis in the right and not the left carotid.

Blumenfeld H. Neuroanatomy Through Clinical Cases, 1st ed. Sunderland, MA: Sinauer Associates; 2002.

31. b

This patient has a medial medullary syndrome with a lesion on the left side. This syndrome is caused by occlusion of the vertebral artery or one of its medial branches, producing an infarct affecting the pyramid, medial lemniscus, and emerging hypoglossal fibers. The patient will have contralateral arm and leg weakness sparing the face (from corticospinal tract involvement prior to its decussation), contralateral loss of sensation to position and vibration, and ipsilateral tongue weakness.

Neither a lateral medullary syndrome, nor a pontine infarct will produce these clinical manifestations.

Goetz CG. Textbook of Clinical Neurology, 3rd ed. Philadelphia, PA: Saunders Elsevier; 2007.

Ropper AH, Samuels MA. Adams and Victor’s Principles of Neurology, 9th ed. New York, NY: McGraw-Hill; 2009.

32. e

This patient may have temporal arteritis or giant cell arteritis, and given the visual manifestations, she should be treated with steroids as soon as possible.

Giant cell arteritis is a disease seen in older adults, typically older than 50 years of age. It is characterized by inflammation of the temporal artery predominantly but may also affect other branches of the ECA. These patients complain of headaches, associated with generalized constitutional symptoms, jaw claudication, and tenderness of the scalp around the temporal artery. This condition may overlap with polymyalgia rheumatica, and patients will also present with proximal muscle pain and achiness. Laboratory studies demonstrate leukocytosis and very elevated sedimentation rates and C-reactive protein levels. The diagnosis is based on a biopsy of the temporal artery demonstrating granulomatous inflammation.

Blindness may occur from ocular ischemia, and these patients should be treated as soon as possible with steroids while arranging for a temporal artery biopsy.

Cerebral angiogram and CT angiogram are not helpful in the diagnosis of this condition. Anticoagulation is not indicated.

Ropper AH, Samuels MA. Adams and Victor’s Principles of Neurology, 9th ed. New York, NY: McGraw-Hill; 2009.

33. a

This patient has a Millard-Gubler syndrome, which is manifested by contralateral hemiplegia with ipsilateral facial palsy. The lesion is localized in the pons, and affects the corticospinal tract before its decussation at the level of the pyramids, as well as the VII cranial nerve nuclei. When there is also conjugate gaze paralysis toward the side of the brainstem lesion it is called Foville syndrome.

Infarcts in the other locations do not cause this constellation of findings.

Ropper AH, Samuels MA. Adams and Victor’s Principles of Neurology, 9th ed. New York, NY: McGraw-Hill; 2009.

Silverman IE, Liu GT, Volpe NJ, et al. The crossed paralyses. Arch Neurol. 1995; 52:635–638.

34. c

Anterior circulation aneurysms have a lower risk of rupture when compared with posterior circulation aneurysms.

Intracranial aneurysms are most often acquired and sporadic; however, there are associations with various conditions, including AVMs, polycystic kidney disease, aortic coarctation, fibromuscular dysplasia, Marfan’s syndrome, and Ehlers-Danlos syndrome. Aneurysms can also be familial.

Cerebral aneurysmal rupture leads to SAH, which is a serious, often fatal event. In a patient with an unruptured cerebral aneurysm, the presence of certain factors promotes a higher risk of rupture and influences treatment decisions. Risk of rupture and SAH depends on various factors. Size is important, and larger aneurysms have a higher risk of rupture, especially those larger than 10 mm. Some studies have suggested that the risk of rupture is significantly higher with diameters of 7 mm or higher; however, this topic is still controversial. Aneurysm location is also important, and posterior circulation aneurysms are at higher risk for rupture than anterior circulation aneurysms. Smoking and uncontrolled hypertension are also risk factors for aneurysmal rupture, and these conditions should be treated. Patients with previous aneurysmal rupture are at higher risk for SAH.

Unruptured aneurysms could be treated conservatively (observation in those patients with low risk of aneurysmal rupture), by endovascular management (coiling), or by surgical management (clipping). Selection of endovascular management or surgical treatment depends on the age of the patient and other comorbidities, size of the aneurysm, location and morphology, as well as the experience of the center where the patient is being treated.

Brisman JL, Song JK, Newell DW. Cerebral aneurysms. N Engl J Med. 2006; 355:928–939.

Wiebers DO, Piepgras DG, Meyer FB, et al. Pathogenesis, natural history and treatment of unruptured intracranial aneurysms. Mayo Clin Proc. 2004; 79:1572–1583.

35. e

Expressive aphasia will not be expected with this infarct. Figure 2.11 shows a left occipital infarct, in the distribution of the left PCA, which will likely cause a homonymous hemianopia in the contralateral side, in this case in the right visual hemifield. Typically, PCA strokes spare central vision because of collateral blood supply to macular cortical representation.

Occipital strokes in the dominant hemisphere may manifest with alexia (inability to read), anomia, achromatopsia (color anomia), and other visual agnosias. Patients with left occipital infarcts involving the splenium of the corpus callosum may present with the classical alexia without agraphia. This syndrome occurs because the patient cannot see what is placed in the right visual hemifield, and whatever can be seen in the left visual hemifield will be represented in the right occipital cortex, but due to corpus callosum involvement, this information will not be transmitted to language centers in the left hemisphere.

Expressive aphasia is caused by lesions in Broca’s area in dominant frontal lobe and is not seen with occipital infarcts.

Ropper AH, Samuels MA. Adams and Victor’s Principles of Neurology, 9th ed. New York, NY: McGraw-Hill; 2009.

36. a

This patient has an infarction in the right mesencephalic tegmentum in its ventral portion, involving the ventral part of the red nucleus, the brachium conjunctivum, and the fascicle of the third cranial nerve. This lesion produces a constellation of findings including an ipsilateral third nerve palsy with contralateral involuntary movements such as tremor and choreoathetosis. The combination of this manifestations have been called Benedikt’s syndrome.

The lesions in the other locations do not cause these clinical manifestations.

Bradley WG, Daroff RB, Fenichel GM, Jankovic J. Neurology in Clinical Practice, 5th ed. Philadelphia, PA: Elsevier; 2008.

37. c

This patient has a left MCA distribution infarct, which in this case is a dominant hemispheric infarct. Dominant hemispheric strokes will manifest with aphasias, depending on the region affected. Superior division MCA strokes predominantly involve the frontal lobe and will manifest with Broca’s aphasia. Inferior division MCA strokes predominantly involve the temporal lobe and will manifest with Wernicke’s aphasia. In this case, given the extent of the lesion, the patient will most likely have a global aphasia.

With MCA strokes, the frontal eye fields may be involved, and patients will have gaze deviation toward the hemisphere involved. This occurs because the contralateral frontal eye fields will be unopposed, “pushing” the eyes to the side of the infarct. Given that optic radiations are also involved during their course within the territory of the MCA, a contralateral homonymous hemianopia is expected. In this case a right homonymous hemianopia.

Patients with MCA strokes present with contralateral hemiparesis. If the stroke predominantly involves the cortex, the weakness will be more prominent in the face and arm than in the leg, as the cortical leg area is supplied by the ACA. If the infarct extends to the subcortical region affecting the corona radiata or internal capsule, the patient could have a dense hemiplegia involving face, arm, and leg equally. Weakness that involves the leg more than the face and arm is characteristic of anterior cerebral infarctions, and not typically seen with MCA infarctions.

Neglect, anosognosia, and other visual-spatial disturbances are seen more frequently with nondominant hemispheric lesions.

Ropper AH, Samuels MA. Adams and Victor’s Principles of Neurology, 9th ed. New York, NY: McGraw-Hill; 2009.

38. c

A 52-year-old patient with hyperthyroidism and intracranial atherosclerotic stenosis should be managed with antiplatelet agents. The other cases should be managed with oral anticoagulation.

Results of the Warfarin-Aspirin Symptomatic Intracranial Disease (WASID) trial suggested that oral anticoagulation with warfarin was associated with more adverse events and provided no benefit over aspirin in the prevention of cerebrovascular events in the setting of intracranial atherosclerotic disease. Use of oral anticoagulation is indicated in the prevention of strokes from a cardioembolic source, and this therapy is not indicated or is controversial in the setting of atherothrombotic etiologies. A patient with intracardiac thrombus should also be treated with anticoagulation. In the setting of anterior wall myocardial infarction with anterior wall akinesis and depressed ejection fraction, the use of anticoagulation should be contemplated, since the risk of formation of intramural thrombus is high in these patients. In general, oral anticoagulation should target an INR between 2.0 and 3.0, except in the setting of mechanical valves, in which the target INR is 2.5 to 3.5.

Treatment with oral anticoagulation should also be considered in patients with atrial fibrillation, and in these cases, the CHADS2 score (discussed in question 58) should be calculated to evaluate if treatment with warfarin is indicated, or if antiplatelet agents would suffice depending on the stroke risk. Other considerations such as risk of hemorrhage (as in patients with high fall risk) should be assessed. Certainly in patients with atrial fibrillation, heart rate control is required, and as suggested by the AFFIRM (Atrial Fibrillation Follow-up Investigation of Rhythm Management) trial, rhythm control does not confer benefit when compared with rate control.

Chimowitz MI, Lynn MJ, Howlett-Smith H, et al. Comparison of Warfarin and Aspirin for symptomatic intracranial arterial stenosis. N Engl J Med. 2005; 352:1305–1316.

Ropper AH, Samuels MA. Adams and Victor’s Principles of Neurology, 9th ed. New York, NY: McGraw-Hill; 2009.

Wyse DG, Waldo AL, DiMArco JP, et al. A comparison of rate control and rhythm control in patients with atrial fibrillation. N Engl J Med. 2002; 347:1825–1833.

39. a

This patient has a stroke from an occlusion of the trunk of the left MCA before the bifurcation, affecting not only the cortex but also the deep subcortical structures provided by lenticulostriate branches.

The hemiparesis affecting face, arm, and leg is explained by involvement of the internal capsule, which is supplied by the lenticulostriate arteries that originate from the stem of the MCA prior to the bifurcation.

An occlusion at this site also explains the cortical findings such as the aphasia. In superior division left MCA strokes, a Broca’s aphasia is more common, whereas Wernicke’s aphasia is seen more often with inferior division left MCA strokes. In this case, the global aphasia is better explained by an MCA trunk occlusion affecting both divisions.

Gaze deviation to the left is caused by unopposed action of the right frontal eye fields. The right homonymous hemianopsia is caused by interruption of the geniculocalcarine radiations running in the left hemisphere.

A stroke localized to the left lenticulostriate branches will not explain the aphasia and the other neurologic findings except the hemiparesis.

An infarct isolated to the individual MCA segments after the first bifurcation will not explain the hemiparesis affecting also the lower extremity, nor the global aphasia.

A pontine stroke will not present with aphasia, and furthermore the eyes will deviate toward the hemiparetic side and not toward the side of the lesion.

Blumenfeld H. Neuroanatomy Through Clinical Cases, 1st ed. Sunderland, MA: Sinauer Associates; 2002.

Goetz CG. Textbook of Clinical Neurology, 3rd ed. Philadelphia, PA: Saunders Elsevier; 2007.

Ropper AH, Samuels MA. Adams and Victor’s Principles of Neurology, 9th ed. New York, NY: McGraw-Hill; 2009.

40. d

The lenticulostriate arteries arise from the trunk of the MCA before its bifurcation.

The circle of Willis is an arterial ring that interconnects the anterior and posterior circulation as well as the right and left arterial systems (Figure 2.21).

FIGURE 2.21 Circle of Willis (Illustration by David R. Schumick, BS, CMI. Reprinted with permission of the Cleveland Clinic Center for Medical Art and Photography. © 2010. All rights reserved.)

The anterior circulation is provided by the ACA and the MCA, which are the terminal branches of the ICA. In the anterior circulation, the right and left sides connect via the anterior communicating arteries. The posterior circulation is constituted by the vertebrobasilar system, with the PCAs being the terminal branches originating from the top of the basilar. The anterior and posterior circulations connect via the posterior communicating arteries.

The ACAs connect through the anterior communicating artery. Beyond this point, the ACA will continue around the genu of the corpus callosum, and it will give rise to various small branches, including the recurrent artery of Heubner, and the pericallosal artery, which runs along the corpus callosum.

The MCA originates from the ICA. The MCA main trunk before the bifurcation gives off many small vessels called the lenticulostriate arteries, which supply large regions of the basal ganglia and the internal capsule. The MCA will bifurcate into superior and inferior divisions that will supply the lateral convexity.

The PCA comes off the top of the basilar, and will run toward the back of the midbrain, along the lateral aspect of the quadrigeminal cistern and around the pulvinar, dividing into smaller branches at the calcarine fissure.

Blumenfeld H. Neuroanatomy Through Clinical Cases, 1st ed. Sunderland, MA: Sinauer Associates; 2002.

Morris P. Practical Neuroangiography, 1st ed. Baltimore, MD: Williams & Wilkins; 1997.

41. a

Statins reduce the risk of cerebrovascular events.

This patient had a TIA with negative cardioembolic work up. The Stroke Prevention by Aggressive Reduction in Cholesterol Levels (SPARCL) trial studied the effect of atorvastatin at a dose of 80 mg daily in patients with a recent (within 6 months) TIA or stroke, with low density lipoprotein (LDL) between 100 and 190 mg/dL. The conclusion was that 80 mg of atorvastatin daily reduces the overall incidence of strokes and cardiovascular events.

Warfarin and heparin are not indicated in this case and are used for the prevention of cardioembolic events. Thrombolysis with tissue plasminogen activator (tPA) is used for the treatment of acute ischemic stroke, but not for prevention of cerebrovascular events. The use of hormone replacement therapy is not indicated, and may be associated with an increased risk of stroke. The severity of stroke may also be increased in patients on hormone replacement therapy.

Amarenco P, Bogousslavsky J, Callahan A III, et al. High-dose atorvastatin after stroke or transient ischemic stroke. N Engl J Med. 2006; 355:549–559.

Bath PM, Gray LJ. Association between hormone replacement therapy and subsequent stroke: a meta-analysis. BMJ. 2005; 330:342.

42. e

This patient has Parinaud’s syndrome, in which there is supranuclear paralysis of eye elevation, defect in convergence, convergence-retraction nystagmus, light-near dissociation, lid retraction, and skew deviation of the eyes. The lesion is localized in the dorsal midbrain, and is classically seen with pineal tumors compressing the quadrigeminal plate; however, it can occur from midbrain infarcts.

Goetz CG. Textbook of Clinical Neurology, 3rd ed. Philadelphia, PA: Saunders Elsevier; 2007.

Ropper AH, Samuels MA. Adams and Victor’s Principles of Neurology, 9th ed. New York, NY: McGraw-Hill; 2009.

43. e

CT scan of the brain showing no evidence of acute infarct is not a contraindication for intravenous tissue plasminogen activator (tPA).

Patients presenting with an acute ischemic stroke within the time window for intravenous tPA should have a brain CT scan to rule out hemorrhage. The CT scan also helps to determine if there are already established signs of ischemia. Evidence of infarcted tissue on CT scan may suggest a longer time from the onset of symptoms than initially thought.

Exclusion criteria for treatment with intravenous tPA in the original trial by the National Institute of Neurological Disorders and Stroke (NINDS) include the following:

– Another stroke or serious head trauma within the prior 3 months

– Major surgery within the prior 14 days

– History of intracranial hemorrhage (ICH)

– Blood pressure above 185/110 mm Hg

– Rapidly improving or minor symptoms

– Symptoms suggestive of SAH

– Gastrointestinal or urinary tract hemorrhage within 21 days

– Arterial puncture at a noncompressible site within 7 days

– Seizure at the onset of the stroke

– Use of anticoagulants

– Patients who received heparin within 48 hours of the onset of stroke and have an elevated activated partial thromboplastin time

– Prothrombin time greater than 15 seconds

– Platelet count below 100,000/mm3

– Glucose less than 50 mg/dL or more than 400 mg/dL

The ECASS3 (discussed in question 3) showed that intravenous tPA, when given between 3 and 4.5 hours after the onset of symptoms, can improve clinical outcomes in patients with acute ischemic stroke. Additional exclusion criteria for this group of patients include National Institutes of Health Stroke Scale (NIHSS) score of 25 or higher, age > 80, any anticoagulant use regardless of INR or prothrombin time, and history of prior stroke and/or diabetes.

Hacke W, Kaste M, Bluhmki E, et al. Thrombolysis with alteplase 3 to 4.5 hours after acute ischemic stroke. N Engl J Med. 2008; 359:1317–1329.

The National Institute of Neurological Disorders and Stroke rt-PA Stroke Study Group. Tissue plasminogen activator for acute ischemic stroke. N Engl J Med. 1995; 335:1581–1587.

44. e

At the time this book is published, aspirin is the only option with evidence to support its use in intracranial stenosis.

The Warfarin-Aspirin Symptomatic Intracranial Disease (WASID) trial randomized patients with a recent TIA or stroke with a 50% to 99% stenosis of a major intracranial artery to warfarin or aspirin, and concluded that warfarin was associated with higher rates of adverse events and provided no benefit over aspirin.

Endovascular procedures have evolved, and angioplasty and intracranial stenting have been used for the treatment of intracranial atherosclerotic stenosis; however, as of 2011, there is no clear evidence to support their use. At the time this book was published, there was an ongoing trial comparing the use of intracranial stent placement versus medical therapy.

Surgical bypass has fallen out of favor since it does not improve clinical outcomes, and patients who undergo such procedures may actually do worse. Surgical bypass may be considered in selected cases.

Chimowitz MI, Lynn MJ, Howlett-Smith H, et al. Comparison of warfarin and aspirin for symptomatic intracranial arterial stenosis. N Engl J Med. 2005; 352:1305–1316.

The EC/IC Bypass Study group. Failure of extracranial-intracranial arterial bypass to reduce the risk of ischemic stroke. Results of an international randomized trial. N Engl J Med. 1985; 313:1191–1200.

45. d

The Alberta Stroke Program Early CT Score (ASPECTS) is calculated based on findings on standard CT scan of the brain and provides a reproducible grading system to assess early ischemic changes in patients with acute ischemic strokes of the anterior circulation. This can be used to guide treatment with intravenous tPA by helping identify patients who will unlikely make an independent recovery.

To obtain the ASPECT score, two axial cuts are obtained on the CT, one at the level of the basal ganglia and thalamus, and another superior cut where these structures are not appreciated. On these sections, there are 10 regions of interest, of which 4 are deep and defined as the caudate, the internal capsule, the lentiform nucleus, and the insular region, and 6 regions are cortical. These regions are assigned a point, which is subtracted if there is early ischemic change in that specific region. A normal looking CT scan will obtain a maximum of 10 points, and a score of 0 is consistent with diffuse ischemic injury of the entire MCA territory.

The ASPECTS correlates inversely with the severity of the stroke, and patients with low scores should not be treated with thrombolytic agents. An ASPECTS score of 7 or less correlates with increased dependence and death.

Warwick Pexman JH, Barber PA, Hill MD, et al. Use of the Alberta Stroke Program Early CT Score (ASPECTS) for assessing CT scans in patients with acute stroke. Am J Neuroradiol. 2001; 22:1534–1542.

46. e

The anterior choroidal artery is a branch of the ICA and arises from the communicating segment.

Various classifications have been proposed for the segments of the ICA, with that by Bouthillier et al., being one of the most widely used and the basis for the following discussion.

The cervical (C1) segment of the ICA, beginning at the level of the common carotid artery and ending where the ICA enters the carotid canal in the petrous bone. It has no branches.

The petrous (C2) segment of the ICA runs within the carotid canal in the petrous bone. Vidian and caroticotympanic branches arise from this segment.

The lacerum (C3) segment of the ICA runs between where the carotid canal ends and the superior margin of the petrolingual ligament. This ligament runs between the lingula of the sphenoid bone and the petrous apex and is a continuation of the periosteum of the carotid canal.

The cavernous (C4) segment begins at the superior margin of the petrolingual ligament, runs within the cavernous sinus, and ends at the proximal dural ring formed by the junction of the medial and inferior periosteum of the anterior clinoid process. Meningohypophyseal trunk, inferolateral trunk, and capsular arteries arise from this segment.

The clinoid (C5) segment runs between the proximal dural ring and the distal dural ring where the ICA becomes intradural. This small segment has no branches.

The ophthalmic (C6) segment begins at the distal dural ring ending proximal to the origin of the posterior communicating artery. Two major branches originate at this level, the ophthalmic artery and the superior hypophyseal artery.

The communicating (C7) segment begins proximal to the origin of the posterior communicating artery extending to the ICA bifurcation. This segment gives off the posterior communicating artery and the anterior choroidal artery.

Blumenfeld H. Neuroanatomy Through Clinical Cases, 1st ed. Sunderland, MA: Sinauer Associates; 2002.

Bouthillier A, van Loveren HR, Keller J. Segments of the internal carotid artery: a new classification. Neurosurgery. 1996; 38:425–433.

47. e

A patient with left carotid stenosis of more than 70% may benefit from carotid endarterectomy (CEA) in this case.

In this case, the right carotid is the one that is producing symptoms, and therefore if a right carotid stenosis is detected, it is symptomatic carotid disease. On the other hand, a left carotid stenosis in this case will be asymptomatic.

Patients with extracranial ICA atherosclerotic disease can be treated medically or surgically to prevent cerebrovascular events. One of the most important factors in deciding if CEA will be beneficial is determination whether a lesion is symptomatic or asymptomatic, along with other patient characteristics.

According to the North American Symptomatic Carotid Endarterectomy Trial (NASCET), in patients with 70% to 99% symptomatic carotid stenosis, the 2-year ipsilateral stroke rate was 26% with medical treatment versus 9% with CEA. Therefore, a symptomatic carotid stenosis of 70% to 99% should be treated surgically. Patients with symptomatic stenosis of 50% to 69% will also benefit from CEA, with greater impact in men versus women, in those with previous strokes versus TIAs, and with hemispheric versus retinal symptoms. In patients with stenosis of less than 50%, there is no evidence that surgical treatment is better than aspirin. The same applies for carotid occlusions, in which the treatment should be medical management.

Regarding asymptomatic carotid disease, CEA has proven benefit over medical treatment in patients with more than 60% stenosis as demonstrated in the Asymptomatic Carotid Atherosclerosis Study (ACAS), and in the Asymptomatic Carotid Surgery Trial (ACST), however, the numbers needed to treat are high, and the benefit may not be significant in the real world, depending also on the experience of the surgeon. In ACAS, the absolute risk reduction was 1.2% per year with a number needed to treat of 85 favoring the surgical group. In the ACST, the absolute risk reduction was 1.1% with a number needed to treat of 93 favoring surgery over medical therapy.

Brott TG, Brown RD, Meyer FB. Carotid revascularization for prevention of stroke: carotid endarterectomy and carotid artery stenting. Mayo Clin Proc. 2004; 79:1197–1208.

48. e

This patient had a TIA, which is a brief episode of focal neurologic dysfunction caused by ischemia without cerebral infarction. It is important to recognize TIAs and evaluate them, since patients who have TIAs are at higher risk of stroke. Around 10% to 15% of patients who suffer a TIA will have a stroke within 3 months, and the risk is higher sooner after the TIA, with 50% of the strokes occurring within 48 hours. Therefore, patients presenting with TIAs should be urgently assessed, with a detailed analysis of their risk factors for stroke. Work up should be obtained to evaluate the vascular origin of the symptoms, to exclude alternative nonischemic causes, and to assess prognostic factors.

The current proposed definition of TIA is based on tissue damage and the presence of infarcted tissue. Brain imaging with MRI and diffusion-weighted imaging (DWI) is helpful in detecting the presence of infracted tissue.

Guidelines recommend the following:

– Neuroimaging within 24 hours, preferably an MRI with DWI

– Noninvasive vascular imaging of the extracranial arteries

– Noninvasive imaging of the intracranial vasculature if it is considered that this will alter management

– Patients should be evaluated as soon as possible.



Given that in the acute presentation a TIA is undistinguishable from an acute stroke, all patients presenting with acute neurologic symptoms should be assessed similarly, and considered candidates for intravenous tPA until proven contraindication is determined. No period of observation should be allowed in this setting, since on acute presentation these two spectrums of the disease cannot be differentiated.

Easton JD, Saver JL, Albers GW, et al. Definition and evaluation of transient ischemic attack. Stroke. 2009; 40:2276–2293.

49. d

Figure 2.13 shows a deep intracerebral hemorrhage, most likely caused by hypertension. Intracranial hemorrhage (ICH) can be primary or secondary (in which it complicates a preexisting lesion). Primary ICH accounts for approximately 10% of all strokes, and is a significant cause of morbidity and mortality. The most common cause of ICH is hypertension which is responsible for 75% of cases, followed by cerebral amyloid angiopathy.

Hypertensive ICH commonly originates in deep subcortical structures such as the putamen, caudate, and thalamus, as well as in the pons, cerebellum, and periventricular deep white matter. This occurs from rupture of deep perforating arteries, which suffer changes caused by chronic hypertension, leading to lipohyalinosis, making these vessels susceptible to sudden closure (causing lacunar infarctions) or rupture (causing hemorrhage).

Amyloid angiopathy is the deposition of congophilic material in the media and adventitia of cerebral vessels, especially cortical and leptomeningeal vessels. This process is associated with cortical and lobar hemorrhages. This condition is seen more commonly in the elderly, and MRI with gradient echo sequences can detect multiple small areas of hemorrhages in the brain, helping to make the diagnosis.

Anticoagulation can be associated with any type of hemorrhage; however, hypertension is a more common cause, and the most likely etiology in this case given the location. Patients on anticoagulation may have hemorrhages showing a fluid-fluid level.

Intracranial aneurysmal rupture presents with SAH rather than intraparenchymal hemorrhage.

Sinus venous thrombosis is associated with hemorrhagic infarcts, usually in the distribution of the thrombosed sinus.

Badjatia N, Rosand J. Intracerebral hemorrhage. Neurologist. 2005; 11:311–324.

Manno EM, Atkinson JL, Fulgham JR, Widjdicks EFM. Emerging medical and surgical management strategies in the evaluation and treatment of intracerebral hemorrhage. Mayo Clin Proc. 2005; 80:420–433.

50. e

Treatments of Moyamoya disease have not been satisfactory, and anticoagulation has not proven to be of benefit, being usually avoided given the hemorrhagic risk in these patients.

Moyamoya is a noninflammatory, nonatherosclerotic vasculopathy that affects the intracranial circulation, leading to arterial occlusions and prominent arterial collateral circulation. It presents most commonly in children and adolescents, with a second peak in the fourth decade of life, but in a much lower frequency. Clinical manifestations include TIAs and strokes, as well as intracranial hemorrhages (ICHs). In childhood the presentation is predominantly ischemic, with strokes and TIAs, which may be precipitated by hyperventilation. In adults, the presentation is most frequently ICH. Other manifestations seen in Moyamoya disease include headaches, seizures, movement disorders, and mental deterioration.

The diagnosis is based on the neuroangiographic findings, characterized by progressive bilateral stenosis of the distal internal carotid arteries, extending to proximal ACAs and MCAs, and the development of extensive collateral circulation at the base of the brain, with the “puff of smoke” appearance. Histopathologically, there is intimal thickening by fibrous tissue of the affected arteries, with no inflammatory cells or atheromas.

There is no curative treatment for this condition. Revascularization procedures may improve perfusion, angiographic appearance, and ischemic manifestations; however, they may not impact the frequency of hemorrhagic events.

Medications such as antiplatelets, vasodilators, calcium channel blockers, and steroids have been used with equivocal results. Anticoagulation is not helpful and usually avoided given the hemorrhagic complications.

Bradley WG, Daroff RB, Fenichel GM, Jankovic J. Neurology in Clinical Practice, 5th ed. Philadelphia, PA: Elsevier; 2008.

Ropper AH, Samuels MA. Adams and Victor’s Principles of Neurology, 9th ed. New York, NY: McGraw-Hill; 2009.

51. a

This patient has a lobar hemorrhage likely associated with amyloid angiopathy. MRI of the brain shown in Figure 2.14 demonstrates multiple small rounded areas of gradient echo susceptibility throughout the brain, representing microhemorrhages with hemosiderin deposition.

Amyloid angiopathy is a condition that causes deposition of amyloid material in the cerebral vessels, especially in the cortex and leptomeninges, and is associated with lobar hemorrhages in the elderly. Since the angiopathy is diffuse, there are recurrent and multiple hemorrhages, and commonly MRI with gradient echo sequences will show multiple small areas of hypointensity suggesting prior hemosiderin deposition from prior hemorrhages, such as in this case.

Histologically, there is deposition of Congo-red positive amyloid material in the media and adventitia of small- and medium-sized vessels. This causes weakening of the vessel walls. There are associations of amyloid angiopathy with apolipoprotein Ε4 and Ε2, as well as with Alzheimer’s disease.

Hypertension does not produce this pattern of microhemorrhages seen on gradient echo sequences.

Intracranial aneurysmal rupture presents with SAH rather than intraparenchymal hemorrhage.

Anticoagulation can be associated with any type of hemorrhage; however, the hemorrhage locations and clinical characteristics of this patient are more consistent with amyloid angiopathy. Patients with amyloid angiopathy are more susceptible to have intracranial hemorrhage (ICH) in the setting of anticoagulation.

Sinus venous thrombosis is associated with hemorrhagic infarcts, usually in the distribution of the thrombosed sinus.

Bradley WG, Daroff RB, Fenichel GM, Jankovic J. Neurology in Clinical Practice, 5th ed. Philadelphia, PA: Elsevier; 2008.

Ropper AH, Samuels MA. Adams and Victor’s Principles of Neurology, 9th ed. New York, NY: McGraw-Hill; 2009.

52. e

This patient has Dejerine-Roussy syndrome, caused by a thalamic infarct, in which the lesion affects the sensory relay nuclei. These patients present with severe deep and cutaneous sensory loss of the contralateral hemibody, usually the entire hemibody and up to the midline. In some cases, there may be dissociation of sensory loss, affecting more pain and temperature sensation than touch, vibration, or proprioception. With time, some sensation returns, but the patient may develop severe pain, allodynia, and paresthesias of the affected body part.

Infarcts in the other structures will not produce the clinical manifestations depicted in this case.

Goetz CG. Textbook of Clinical Neurology, 3rd ed. Philadelphia, PA: Saunders Elsevier; 2007.

Ropper AH, Samuels MA. Adams and Victor’s Principles of Neurology, 9th ed. New York, NY: McGraw-Hill; 2009.

53. b

The anterior choroidal arteries arise from the anterior circulation, more specifically from the ICA. The posterior choroidal arteries arise from the posterior circulation, more specifically from the posterior cerebral arteries.

The vertebral arteries originate from the subclavian arteries on their respective sides. The V1 segment extends from the subclavian artery to the transverse foramen of C5-C6. The V2 segment runs within the transverse foramina of the cervical vertebra from C5-C6 to C2. The V3 segment extends from the transverse foramen of C2 and turns posterolaterally around the arch of C1, between the atlas and the occiput. This segment is extracranial. The V4 segment begins where the vertebral artery enters the dura at the foramen magnum and joins the contralateral vertebral artery to form the basilar artery. The vertebral artery, at the V4 segment gives off the posterior inferior cerebellar artery (PICA) and the anterior spinal artery. Both vertebral arteries will join to form the basilar artery. The posterior circulation provides the vascular supply to the brainstem, cerebellum, the thalamus, and the occipital lobes.

Blumenfeld H. Neuroanatomy Through Clinical Cases, 1st ed. Sunderland, MA: Sinauer Associates; 2002.

Shin JH, Suh DC, Choi CG, et al. Vertebral artery dissection: spectrum of imaging findings with emphasis on angiography and correlation with clinical presentation. RadioGraphics. 2000; 20:1687–1696.

54. d

Based on the history and MRI findings in Figure 2.15, this patient most likely has a hemorrhagic infarct from sinus venous thrombosis, in this case, a left transverse sinus thrombosis. Occlusion of the venous sinuses will lead to venous infarction and localized edema. The affected tissue becomes engorged, swollen, and the parenchyma will suffer ischemia, leading to infarct and hemorrhage. Given the occlusion of venous drainage and in the setting of parenchymal edema, the content of the intracranial volume will rise, leading to intracranial hyper-tension.

Clinical presentation is characterized by the presence of headache, and depending on the extent of the disease, focal neurologic deficits, altered mental status, seizures, and coma will be present, sometimes progressing to herniation and death.

Risk factors are often encountered including prothrombotic states, either genetic or acquired, such as antiphospholipid antibody syndrome, pregnancy, and the use of oral contraceptives. Other causes include infections (otitis, mastoiditis, sinusitis, meningitis), inflammatory conditions, trauma, dehydration, and neoplastic processes.

The diagnosis should be considered in young patients with prothrombotic risk factors presenting with clinical manifestations suggestive of this condition. CT scan and MRI of the brain will show hemorrhagic infarcts that are not in a strictly arterial distribution (as in Figure 2.15, in which the infarct seems to involve both the left MCA and PCA territories). MRV will demonstrate the absence of signal in the thrombosed venous sinus.

Treatment involves stabilization of the patient and anticoagulation to stop the thrombotic process. Treatment of intracranial hypertension may be needed, sometimes requiring surgical decompression and removal of the hemorrhagic infarct. In some cases, endovascular intervention for thrombolysis and clot removal is required.

This is not an arterial infarct since it does not follow the distribution of an arterial vascular territory. This patient does not have a history of hypertension, and the characteristics of the MRI are not typical of hypertensive hemorrhage. This patient is not in the age group for amyloid angiopathy-related intracranial hemorrhage, and the history suggests an alternative diagnosis such as venous sinus thrombosis. The hemorrhage in this patient is not in the subarachnoid space.

Ropper AH, Samuels MA. Adams and Victor’s Principles of Neurology, 9th ed. New York, NY: McGraw-Hill; 2009.

Stam J. Thrombosis of the cerebral veins and sinuses. N Engl J Med. 2005; 352:1791–1798.

55. a

The use of warfarin is less warranted based on the available evidence. The Warfarin-Aspirin Symptomatic Intracranial Disease (WASID) trial randomized patients with a recent TIA or stroke with a 50% to 99% stenosis of a major intracranial artery to warfarin or aspirin. The primary end point was ischemic stroke, intracranial hemorrhage (ICH), or death from vascular causes other than stroke. At the end of the study, it was concluded that warfarin was associated with higher rates of adverse events and provided no benefit over aspirin in these patients.

Antiplatelet agents have been the mainstay of therapy for this patient population, and aspirin was the one assessed in the WASID trial. The American Heart Association/American Stroke Association (AHA/ASA) recommends aspirin monotherapy, aspirin/dipyridamole, or clopidogrel monotherapy as acceptable options for the prevention of noncardioembolic strokes.

On the basis of the Stroke Prevention by Aggressive Reduction in Cholesterol Levels (SPARCL) trial, the AHA/ASA recommends statin therapy for patients with atherosclerotic ischemic stroke and TIA, to reduce the risk of subsequent cerebrovascular events.

Amarenco P, Bogousslavsky J, Callahan A III, et al. High-dose atorvastatin after stroke or transient ischemic stroke. N Engl J Med. 2006; 355:549–559.

Chimowitz MI, Lynn MJ, Howlett-Smith H, et al. Comparison of warfarin and aspirin for symptomatic intracranial arterial stenosis. N Engl J Med. 2005; 352:1305–1316.

Qureshi AI, Feldmann E, Gomez CR, et al. Intracranial atherosclerotic disease: an update. Ann Neurol. 2009; 66:730–738.

Sacco RL, Adams R, Albers G, et al. Guidelines for prevention of stroke in patients with ischemic stroke or transient ischemic attack. Stroke. 2006; 37:577–617.

56. c

This patient has Claude’s syndrome, manifested by ipsilateral third nerve palsy, and contralateral ataxia and tremor. The lesion affects the dorsal red nucleus and the third nerve fascicle and is in the midbrain tegmentum more dorsally located than the lesion seen in Benedikt’s syndrome, which is caused by a ventral mesencephalic tegmental lesion. Patients with Claude’s syndrome, as compared to Benedikt’s syndrome, have more ataxia but no involuntary choreoathetotic movements.

Lesions in the other locations will not present with the clinical features depicted in this case.

Bradley WG, Daroff RB, Fenichel GM, Jankovic J. Neurology in Clinical Practice, 5th ed. Philadelphia, PA: Elsevier; 2008.

Goetz CG. Textbook of Clinical Neurology, 3rd ed. Philadelphia, PA: Saunders Elsevier; 2007.

57. c

This patient has a left transverse sinus thrombosis, as seen in the MRV in Figure 2.16, in which the signal is lost in the left transverse sinus.

In patients with venous sinus thrombosis, an etiologic factor should be investigated, since it may be a treatable condition. Possible causes include oral contraceptives, pregnancy and puerperium, cancer, nephrotic syndrome, antiphospholipid syndrome, connective tissue disorders, hematologic conditions, trauma, and genetic prothrombotic conditions such as protein C and S deficiency, antithrombin deficiency, factor V Leiden mutation, prothrombin mutation, and homocysteinemia. Infectious causes may also lead to thrombosis in nearby venous structures, and middle ear infections as well as mastoiditis have been associated with transverse sinus thrombosis.

Diabetes and hypertension are typical risk factors for arterial strokes, but further search for prothrombotic risk factors should be done in cases of venous thrombosis. Since this is a venous thrombosis and not an arterial stroke, an embolic source does not need to be investigated, and therefore, an echocardiogram with bubble study and Holter monitor are not required.

In sinus venous thrombosis, anticoagulation is important to stop the thrombotic process. In cerebral venous thrombosis, studies comparing anticoagulation with placebo have shown no increased or new cerebral hemorrhages with anticoagulation, even in patients with the presence of hemorrhagic infarcts before the treatment.

Endovascular thrombolysis with pharmacological agents or mechanical clot removal can be performed but should be restricted to patients with severe neurologic impairment. However, there is no clear evidence comparing the results of this therapy with anticoagulation.

Ropper AH, Samuels MA. Adams and Victor’s Principles of Neurology, 9th ed. New York, NY: McGraw-Hill; 2009.

Stam J. Thrombosis of the cerebral veins and sinuses. N Engl J Med. 2005; 352:1791–1798.

58. e

Renal disease is not a risk factor routinely taken into account for the assessment of stroke risk in patients with TIA.

Patients with atrial fibrillation are at risk for ischemic stroke; however, this population is heterogenous and the risk may vary depending on other risk factors. It is known that warfarin is beneficial to prevent strokes in patients with atrial fibrillation, especially when the stroke risk is high. However, the use of aspirin may be favored for low-risk patients. This is one of the reasons why the estimation of stroke risk in atrial fibrillation is important. The CHADS2 score has been validated for this purpose, in which 1 point is assigned for the presence of each of the following: congestive heart failure, history of hypertension, age of 75 years or older, diabetes mellitus, prior stroke, or TIA. Two points will be given for a history of stroke or TIA. The total score will range from 0 to 6 and will correlate with an expected stroke rate per 100 patient-years, with higher CHADS2 scores correlating with higher stroke rates.

This patient is aged 67 years, with history of diabetes and hypertension, and a recent TIA. All these factors will give him a CHADS2 score of 4, and therefore the stroke rate per 100 patient-years for this patient is approximately 8.5.

Gage BF, Waterman AD, Shannon W, et al. Validation of clinical classification schemes for predicting stroke. Results from the National Registry of Atrial Fibrillation. JAMA. 2001; 285:2864–2870.

59. d

In the brain, drainage of venous blood occurs through veins into dural venous sinuses, which eventually drain into the internal jugular veins. The dural venous sinuses are venous channels enclosed between dural layers, and they have no valves. The major venous sinuses are the superior sagittal sinus, the inferior sagittal sinus, the straight sinus, the transverse sinus, sigmoid sinus, and the cavernous sinus (Figure 2.22).

The superior sagittal sinus runs along the superior margin of the falx cerebri in the interhemispheric fissure, toward the confluence of the sinuses where it encounters the straight sinus and the transverse sinuses, which will continue as the sigmoid sinuses, draining into the internal jugular veins. The inferior sagittal sinus runs above the corpus callosum in the interhemispheric fissure.

The deep venous system is composed of the internal cerebral veins, which will empty into the great cerebral vein of Galen. The basal vein of Rosenthal is a deep vein that drains the base of the forebrain, and also empties into the great cerebral vein of Galen, located beneath the splenium of the corpus callosum. The great cerebral vein of Galen joins the inferior sagittal sinus at the straight sinus, which will then run toward the confluence of the sinuses.

FIGURE 2.22 Venous sinuses (Illustration by David R. Schumick, BS, CMI. Reprinted with permission of the Cleveland Clinic Center for Medical Art and Photography. © 2010. All rights reserved)

The cavernous sinuses are on both sides of the sella turcica and receive blood from facial and orbital structures, including the ophthalmic veins. The cavernous sinuses drain into the superior and inferior petrosal sinuses, which then drain into the sigmoid sinus.

The convexity of the brain has multiple superficial veins that drain into the superior sagittal sinus, the transverse sinus, or the middle cerebral vein, which runs along the Sylvian fissure. Between these structures there are anastomotic veins: the superior anastomotic vein of Trolard and the inferior anastomotic vein of Labbe.

Emissary veins connect scalp veins with the dural venous sinuses.

Blumenfeld, H. Neuroanatomy Through Clinical Cases, 1st ed. Sunderland, MA: Sinauer; 2002.

Morris P. Practical Neuroangiography, 1st ed. Baltimore, MD: Williams & Wilkins; 1997.

60. c

This patient has cerebral autosomal dominant arteriopathy with subcortical infarcts and leukoencephalopathy (CADASIL), which can be suspected based on the history of multiple strokes without traditional vascular risk factors, a history of migraines, subsequent development of dementia, and a family history of strokes and dementia suggesting a hereditary condition. The diagnosis is confirmed with the histopathologic specimen shown in Figure 2.17, demonstrating a blood vessel with a thick wall, which contains a basophilic granular material. This pathologic finding is characteristic of CADASIL.

This condition is associated with a mutation in the gene NOTCH3 on chromosome 19. The gene product is a transmembrane receptor expressed mainly in vascular smooth muscle, and the mutation leads to accumulation of this protein in the vascular walls, especially in small arteries and capillaries.

CADASIL is inherited in an autosomal dominant fashion, and patients present with migraines with aura, stroke episodes, seizures, pseudobulbar palsy, and progressive cognitive decline leading to the development of dementia. The strokes are recurrent and predominantly lacunes, caused by small vessel disease. Some patients present with psychiatric manifestations, especially depression and emotional lability. Parkinsonism is not a typical feature of CADASIL.

The MRI typically shows T2 hyperintensities in the subcortical white matter and basal ganglia. The diagnosis could be made based on detection of the NOTCH3 mutation, or with skin biopsy demonstrating pathologic changes.

Currently, there is no treatment for this condition.

Bradley WG, Daroff RB, Fenichel GM, Jankovic J. Neurology in Clinical Practice, 5th ed. Philadelphia, PA: Elsevier; 2008.

61. b

This patient has a Weber’s syndrome, which is a combination of an ipsilateral third nerve palsy with contralateral hemiplegia. This is caused by a midbrain lesion, in this case a left midbrain infarct.

Brainstem infarcts manifest with crossed syndromes, in which there are ipsilateral cranial nerve abnormalities and contralateral long tract signs.

This patient has a left third nerve palsy, manifested by limited adduction and upward movements of the left eye, therefore presenting with diplopia on upward gaze and when looking to the right side. The contralateral hemiplegia is caused by the lesion affecting the corticospinal tract before its decussation at the level of the pyramids, in this case at the cerebral peduncle. An ipsilateral third nerve palsy with contralateral hemiplegia localizes the lesion to the midbrain.

Bradley WG, Daroff RB, Fenichel GM, Jankovic J. Neurology in Clinical Practice, 5th ed. Philadelphia, PA: Elsevier; 2008.

Goetz CG. Textbook of Clinical Neurology, 3rd ed. Philadelphia, PA: Saunders Elsevier; 2007.

62. b

Intracranial vascular malformations include arteriovenous malformations (AVMs), cavernous malformations, venous angiomas, capillary telangiectasias, and dural arteriovenous fistulas (DAVF).

Cavernous malformations are clusters of vascular channels, composed of dilated thin-walled vessels, with no smooth muscle or elastic fibers, and with no intervening brain parenchyma separating the vascular structures. On MRI, they have the typical “popcorn-like” appearance, with a dark rim on T2 consistent with hemosiderin; gradient echo images (see Figure 2.18) may show evidence of cavernous malformations when not evident on T2-weighted images.

AVMs are congenital vascular lesions in which arteries and veins communicate without an intervening normal capillary bed in between. This lesion can be seen on CT, and CT angiogram (CTA) provides better vascular visualization. MRI demonstrates the vascular lesion with flow voids and regions of previous hemorrhage, as well as its relationship with the parenchyma. Conventional angiography is the standard to evaluate the vascular structure, pattern of vascular feeders, and drainage.

Venous angiomas are thin-walled venous structures with normal intervening brain tissue. These are asymptomatic with a very low risk of hemorrhage. MRI demonstrates a conglomerate of vessels in a “caput medusae pattern.”

Capillary telangiectasias are abnormally dilated capillaries that are separated by normal brain tissue. They are typically found incidentally and rarely become symptomatic.

This is not a DAVF (discussed in question 64).

Bradley WG, Daroff RB, Fenichel GM, Jankovic J. Neurology in Clinical Practice, 5th ed. Philadelphia, PA: Elsevier; 2008.

Brown RD, Flemming KD, Meyer FB, et al. Natural history, evaluation and management of intracranial vascular malformations. Mayo Clin Proc. 2005; 80:269–281.

Prayson RA, Goldblum Jr. Neuropathology, 1st ed. Philadelphia, PA: Elsevier; 2005.

63. e

The common carotid artery ascends in the neck and divides into external and internal carotid arteries at the level of C4, below the angle of the jaw.

The arterial supply to the brain is divided into two main territories, the anterior and posterior circulation. The two internal carotid arteries provide perfusion to the anterior circulation, and the two vertebral arteries provide perfusion to the posterior circulation.

The aortic arch has three main branches, the innominate (brachiocephalic) artery, the left common carotid artery, and the left subclavian artery. Several normal variants exist in both the extracranial and intracranial circulation; the most common anatomy is described below.

The right ICA originates from the right common carotid artery, which is a branch of the innominate artery that comes off the aortic arch. The right vertebral artery comes off the right subclavian artery, which also branches off the innominate artery, and eventually joins the left vertebral artery to form the basilar artery.

The left common carotid artery originates directly from the aortic arch, and divides into left internal and left external carotid arteries. A small percentage of the population have what is called a “bovine aortic arch,” in which the left common carotid has the same origin as the innominate artery, and in some cases the left common carotid will branch off the innominate artery.

The left subclavian artery originates from the aortic arch and gives off the left vertebral artery, which then ascends in the neck to join the right vertebral artery to form the basilar artery.

Blumenfeld H. Neuroanatomy Through Clinical Cases, 1st ed. Sunderland, MA: Sinauer Associates; 2002.

Ropper AH, Samuels MA. Adams and Victor’s Principles of Neurology, 9th ed. New York, NY: McGraw-Hill; 2009.

64. c

This patient has a spinal dural arteriovenous fistula (DAVF), which is the most common type of spinal vascular malformation. This lesion can be fed by one or multiple arteries and is a low pressure and low flow vascular lesion. DAVF is more frequent in men and above 50 years of age. The typical location is in the lower thoracic and lumbar regions, and this type of lesion rarely produces hemorrhage. Patients present with pain, weakness, and sensory symptoms below the level of the lesion. The myelopathic syndrome is usually gradually progressive and caused by venous hypertension and congestion. MRI of the spine shows enlargement of the cord with hyperintensity on T2 seen over several levels, with intradural flow voids suggesting this pathology (Figure 2.19). Spinal angiogram is the gold standard diagnostic test to detect this abnormality. Treatment involves angiographic embolization and/or surgical removal of the lesion. The radiographic findings in this case are consistent with an expanding lesion with T2 hyperintensity in the cord. The presentation is gradually progressive and consistent with DAVF.

Epidural hematoma and spinal cord infarct have a more acute presentation and do not show the MRI findings seen in this case. This is not a cavernous malformation or a venous angioma, which are not common in this location.

Bradley WG, Daroff RB, Fenichel GM, Jankovic J. Neurology in Clinical Practice, 5th ed. Philadelphia, PA: Elsevier; 2008.

65. d

This patient has intracranial hemorrhage (ICH) in the right anterior temporal lobe as seen in the brain CT as a hyperdense area (Figure 2.20). The MRI can be helpful to estimate the timing of the hemorrhage on the basis of the characteristics of the blood and the composition. Initially hemorrhage is composed of plasma and red blood cells, with the presence of intracellular oxyhemoglobin, which is then converted to deoxyhemoglobin, subsequently oxidized to methemoglobin. Later, cell lysis occurs with the presence of extracellular hemoglobin, which is then converted to hemosiderin.

Estimation of the timing of the hemorrhage is based on the presence of these different types of blood products, and is as follows:

– Between 0 and 24 hours: The predominant blood product is oxyhemoglobin and is seen on MRI as isointense on T1 sequences, and hyperintense on T2 sequences.

– Between 1 and 3 days: Deoxyhemoglobin is the predominant blood product, seen as isointense on T1 and hypointense on T2.

– Between 3 and 7 days: Intracellular methemoglobin, seen as hyperintense on T1 and hypointense on T2.

– One week up to probably a month: There is extracellular methemoglobin, seen as hyperintense on T1 and T2 sequences.

– After 1 or 2 weeks, and probably for years: There is hemosiderin, seen as isointense or hypointense on T1, and hypointense on T2 sequences.

Based on these estimates, this patient’s hemorrhage is approximately 1 week old.

Bradley WG, Daroff RB, Fenichel GM, Jankovic J. Neurology in Clinical Practice, 5th ed. Philadelphia, PA: Elsevier; 2008.



If you find an error or have any questions, please email us at admin@doctorlib.org. Thank you!